Ateista Klub

“Az írástudók és a farizeusok a Mózes székében ülnek: Annakokáért a mit parancsolnak néktek, mindazt megtartsátok és megcselekedjétek; de az ő cselekedeteik szerint ne cselekedjetek. Mert ők mondják, de nem cselekszik.” Máté 23:2-3 "ne figyeljenek oda arra, amit mondok, egyetlen dologra figyeljenek, amit csinálok" Orbán Viktor

Friss hozzászólások

Címkék

1 (1) abortusz (2) Ádám és Éva (2) adó (1) agnoszticizmus (9) agresszió (4) AIDS (1) áldozat (4) alkotmány (1) államegyház (6) állatvédők (1) altruizmus (2) áltudomány (3) Amerika (1) analitikus (2) analógia (1) anarchizmus (2) anglia (1) anglikán egyház (1) angyalok (1) animizmus (1) antiszemitizmus (1) antropocentrizmus (2) argumentum ad ignorantiam (1) ateista (10) ateisták (1) ateista egyház (4) ateista párt (1) ateizmus (25) ausztria (1) az ateizmus nem hit (6) a hit ereje (2) a vallások vége (11) a vallás bűnei (2) a vallás vége (8) babona (1) bátorság (2) bayer (1) béke (3) berg (1) bergoglio (3) bertrand russel (1) betegség (9) biblia (11) Biblia (16) bizalom (1) bloggolás (1) boko haram (1) boldog (1) boldogság (10) bolgogság (1) börtön (2) boszorkányüldözés (2) botrány (1) breivik (2) búcsúcédulák (1) buddhizmus (12) bújkáló isten (2) bűnkultusz (2) bűnök (13) bűnözés (37) burka (1) bűvészet (1) cáfolás (1) carl sagan (1) celebek (1) cenzúra (7) cherry picking (1) család (1) csillaghamu (1) csoda (11) csodák (1) dawkins (4) deizmus (7) dekadencia (1) demarkáció (3) demográfia (1) demokrácia (6) Dennett (1) descartes (8) diderot (5) divergencia (16) djihad (2) dogma (1) douglas adams (1) dőzsölés (1) drogok (2) dualizmus (10) dzsihád (2) egészség (1) egyenlőség (2) Egyesült Államok (1) egyház (3) egyházadó (1) egyházállam (20) egyházkritika (5) egyháztörvény (3) egyiptom (1) egzaktság (1) egzisztencializmus (2) eincheitswissenschaft (2) életfilozófia (2) életrajz (2) életszemlélet (5) élet értelme (26) eliminativizmus (1) ellenőrzés (2) ellentmondások (2) elmefilozófia (3) elmélet (1) élmény (3) elnyomás (3) elv (1) empirizmus (7) eq (1) eretnekek (3) erkölcs (29) erkölcsi relativizmus (7) erőszak (11) erotika (2) értékrend (3) értelem (5) értelem és érzelem (8) érvelési hiba (3) érzelem (7) esztétika (3) etika (26) etikaoktatás (2) etikaóra (6) etiopia (1) eu (1) eucharistia (1) evangelium (1) evangéliumok (8) evolúció (13) evolúcó (1) ezotéria (5) facebook (2) fanatizmus (8) fejlődés (1) feltámadás (4) felvilágosodás (6) feminizmus (5) fenyő (1) fetisizmus (1) feymann (1) film (16) filozófia (1) filozófiai racionalizmus (1) filozófus (17) filozófusok (4) finnország (2) finomhangoltság (5) fizika (3) fizikalizmus (4) flow (4) fogalomrendszer (1) függőség (1) fundamentalizmus (6) genezis (1) globalizáció (1) gonosz (21) gy (1) gyerekek (3) gyilkosság (3) gyónás (1) háború (1) hadisz (1) hadith (1) hagyomány (1) halál (19) halál közeli élmény (2) házasság (2) hazugság (1) hedonizmus (1) Heidegger (2) hézagok istene (1) higiénia (1) himnusz (1) hinduizmus (8) hinduk (1) hírcsárda (1) hit (20) hitchens (3) hittan (4) hitvi (1) hitvita (3) hit és tudás (16) hit nélkül élni (8) homeopátia (1) homoszexualitás (8) Hume (2) humor (21) húsvét (1) idealizmus (3) időutazás (1) igazi vallás (1) igazolás (23) igazságosság (2) ikon (1) ima (4) india (5) indonézia (2) indukció (1) inkvizíció (15) instrumentalizmus (2) integráció (1) intellektuális tisztesség (2) intelligencia (4) intelligens tervezés (1) intolerancia (3) irán (1) Irán (1) irodalom (2) irónia (3) irracionalitás (1) isten (11) istenérv (22) Isteni Téveszme (1) istenkáromlás (2) isten halott (1) isten nélkül nincs erkölcs (2) iszlám (46) ízlés (1) izrael (2) játszmaelmélet (1) jézus (24) Jézus (11) jog (1) jóságosság (1) kálvinizmus (2) karácson (1) karácsony (8) karikatúra (1) katasztrófa (1) katolicizmus (11) katolikusok (2) kdnp (3) kereszt (1) keresztelés (1) keresztény (9) keresztényésg (3) kereszténység (38) keresztes hadjáratok (1) kettős mérce (1) Kierkegaard (1) kínzás (6) kivégzés (1) klerikalizmus (2) kognitív disszonancia (1) kölcsey (1) kommunizmus (4) kontinentális filozófia (1) könyv (53) könyvégetés (1) korán (3) koron (1) koronavírus (1) korrupció (1) körülmetélés (3) középkor (6) kozmológia (2) közösség (1) kreacionizmus (7) kreacionmizus (1) kult (1) kultúra (2) legenda (1) lélek (9) lengyelország (1) liberalizmus (3) librivox (1) logika (4) lopás (2) lövöldözés (1) luther (2) magyarázat (2) maher (1) mali (1) mária (2) mártírok (1) maslow (1) matematik (1) matematika (2) materializmus (10) matterhorn (1) mazochizmus (1) medicína (1) meditáció (1) megbocsátás (1) megtermékenyítés (2) mémelmélet (2) menekültkérdés (3) mennyország (12) mérleg (1) mese (3) mesterséges intelligencia (6) metafizik (1) metafizika (11) metafóra (6) metodika (1) militantizmus (1) mise (1) miszt (1) miszticizmus (1) mitológia (2) mítosz (5) modernizáció (2) módsze (1) módszertan (2) monizmus (2) monoteizmus (3) moore (1) mormonizmus (1) mormonok (1) multikulti (3) muszlim (2) mutyi (1) művészet (2) múzeum (1) náci (1) nácizmus (4) naturalizmus (1) NDE (1) németország (4) nepál (1) népek ópiuma (10) népírtás (4) népszámlálás (1) neurobiológia (5) neurózis (2) nevelés (1) nietzsche (4) nobel (1) nők (1) objektív (6) objektív és szubjektív (5) occam (5) okság (1) oktatás (11) ökumenizmus (1) öncsonkítás (1) öngyilkosság (2) önszerveződés (1) ontológia (3) örök élet (2) orvoslás (1) ősrobbanás (1) összehasonlító valláskritika (1) pál (2) palesztína (1) panteizmus (2) pap (1) pápa (4) paradoxon (1) paranoia (3) pascal (3) pedofilia (1) pedofília (3) plágium (1) pogányság (3) pogrom (1) pokol (3) politeizmus (2) politika (19) pornó (2) pozitivizmus (1) predesztináció (1) prostitúció (1) provokáció (2) prüdéria (2) pszichedelikus (1) pszichológia (8) qualia (2) rabszolgaság (2) racionalizmus (7) radikalizmus (1) ratzinger (2) redukcionimzus (2) redukcionizmus (2) reform (1) reformáció (2) regresszió (1) reinkarnáció (3) rejtőzködő isten (2) relativizmus (3) remény (1) reprodukálhatóság (1) repülő (1) Richard Dawkins (2) rossz gyógyszer (2) saeed malekpour (1) sajtószabadság (4) sartre (1) sátán (2) satyagraha (1) sci (1) sci-fi (2) skizofrénia (1) sorozat (2) spagettiszörny (2) spiritualizmus (4) statisztika (13) Sunday Assembly (3) svájc (1) szabadság (8) szabad akarat (7) szadizmus (3) szaturnália (1) szegénység (1) szekta (2) szekták (4) szekularimzus (4) szekularizmus (40) szemet szemért (1) szent könyv (2) szent tehén (1) szerelem (2) szeretet (6) szex (5) szimuláció (4) szintetikus (2) szintetikus apriori (1) szkepticizmus (2) szólásszabadság (2) szollipszizmus (1) sztoicizmus (1) szub (1) szubjektív (7) szüzesség (1) szűznemzés (2) takonyangolna (1) talmud (1) tanmese (22) tanulás (1) taoizmus (1) társadalom (6) tautológia (1) TED (1) teizmus (1) tekintély (1) tény (1) teodícea (1) teodicea (8) teológia (10) teremtés (2) teremté ember az istent (1) természet (1) természettörvények (3) terroizmus (1) terrorizmus (17) tervezés (1) test és elme (4) tinik (1) tízparancsolat (4) tolerancia (2) történelem (10) történelmi jézus (6) transzcendencia (2) transzcendens (4) tudás (2) tudatosság (2) tudomány (21) tudományfilozófia (34) túlvilág (11) tüntetés (2) tv (1) üdvtörténet (1) újságírás (2) újtestamentum (3) üldözés (5) undefined (2) unitárianizmus (1) Univerzum (5) usa (4) USA (2) utópia (1) üzletegyház (1) vagyon (1) vágyvezérelt gondolkodás (6) vakok országa (3) válás (1) vallás (24) vallásfesztivál (1) vallásháború (7) valláskritika (5) vallások vége (5) vallásszabadság (18) vallástudomány (1) vallásüldözés (4) vallás haszna (5) valószínűségszámítás (3) vámpírok (1) varázslás (2) vasárnap (2) vatikán (9) vatikáni szerződés (5) végítélet (1) végtelen regresszus (3) véletlen (1) véletlen egybeesés (1) vermes géza (1) vicc (2) videó (6) vikingek (1) világvége (1) vita (2) voltaire (1) vulgáris (1) zavargás (2) zene (3) zombi (1) zsid (1) zsidók (11) zuhanó repülőgép (1) Címkefelhő

e-mail: maxval1967@gmail.com

e-mail: popocatepetl@freemail.hu

e-mail: miigyelunk@gmail.com

Egy nyilvános hitvita hiábavalóságáról

Brendel Mátyás 2013.11.21. 06:04

mezei-szalai.jpgAhogy a képen látható meghívó illusztrálja, nemrég egy nyilvános vita zajlott Szalai Miklós és Mezei Balázs között a Pázmányon, amelyen állítólag sok volt az érdeklődő. Én magam nem tudtam elmenni, az alábbiakban egy kommentelő beszámolóját olvashatjátok. A kommentelő beszámolóját kommentben használt átkötőszöveg törlésén kívül változtatás nélkül adom közre. 
***
A Pázmány Péter Katolikus Egyetem BTK, Filozófia Tanszék szervezte Mezei Balázs és Szalai Miklós vitáját. A hirdetményben az alábbi szerepelt: „A megvitatott kérdések: Van Isten? Mire jó a vallás? Van-e még értelme e kérdéseknek a tudomány korában?” A vitát Bakos Gergely, a Sapientia Szerzetesi Hittudományi Főiskola docense vezette.
A vita – legalábbis számomra – megdöbbentően gyenge volt. Az elején a két vitázó felet az ateista és a teista szemszög képviselőinek mutatták be. Azt gondolom, hogy önmagában az „ateizmust” egy ilyen vitában definíció nélkül hagyni elég nagy hiba (erős vagy gyenge), bár Szalai Miklós az elején kimondta, hogy ő jelenleg csak a három nagy monoteista vallás istene ellen fog érvelni, ilyen értelemben ateista, de a vita során az agnoszticizmus említésre sem került, ami meglepő.
Az első részben mindkét előadó egyfajta alapvetést tett kb. 10 percben, isten létének kérdéséről. Mezei Balázs a „van-e isten?” kérdés jelentéséről elmélkedett, meglehetősen nehezen megfogható módon, ilyesfajta kijelentésekkel: az isten van-ja más, mint a többi van, abszolút és végtelen. Közben azt gondoltam, hogy (legalábbis Carnap alapján) tulajdonképpen semmit nem mondott.
Szalai Miklós először a három ismert ellen-istenérvet hozta fel: a rossz létét, isten rejtőzködését, illetve úgy emlékszem azt, hogy az univerzumban nincs helye istennek. Emellett elmondta, hogy – bár isten és túlvilág nincs szerinte – valamilyen felsőbb teremtő létét nem lehet cáfolni. Ez még nem lenne baj, de ezek után abszolút, objektív morálról és objektív jó-rosszról is beszélt, ami szerintem, erősen megkérdőjelezhető úgy, hogy semmilyen indoklást nem ad arra, hogy ezeket miért fogadja el.
Igazi vita ezek után nem alakult ki, az egész leginkább a vallásosok megnyugtatására szolgáló, felszínes beszélgetésnek látszott. Még egyszer-kétszer cseréltek, de a moderátor és a két vitázó is inkább csak kijelentett, kevéssé érvelt vagy reagált.
A legvégén néhány írásban leadott kérdésre is válaszoltak. A legnagyobb probléma szerintem itt volt. Három kérdést tettem fel:
1) Hogy lehet az agnoszticizmus említése nélkül vitázni isten létéről vagy nem létéről?
2) Egy analitikus filozófus (Szalai Miklós) milyen igazolást fogad el az objektív jó, rossz, morál létére?
3) Mezei Balázs milyen értelemben használja az „abszolút” és „végtelen” fogalmakat isten van-jára?

A következő válaszokat kaptam:
1) Bakos Gergely: Nahát, tényleg, hogy nem?! Miért nem hívtunk meg agnosztikust? Hehehe, hahaha…
2) Erre nem kaptam semmilyen választ. Álválaszt sem, semmi reakciót.
3) Mezei Balázs: Ezeket a fogalmakat nem lehet megmagyarázni, de utána lehet olvasni.

Ez összességében nagyon furcsa volt, bár utólag semmi meglepőt nem látok abban, hogy a vallásosok nem hajlandóak komoly vitát folytatni isten létének a kérdéséről. Már csak azért sem, mert nem nagyon van miről vitázniuk. Úgy tűnt, hogy Mezei leginkább metafizikus úton közelített (szerintem így nem közelít…) a kérdéshez, Szalai pedig nem csapott le az semmiről való beszélés értelmetlenségére.
Az egyébként jó hangulatú beszélgetés során semmilyen érv ütköztetésére nem került sor, például nem hangsúlyozták, hogy jelenlegi ismereteink szerint semmi szükségünk isten létének feltételezésére, illetve egyenrangúan kezelték a teológiai, filozófiai és természettudományos állításokat. Így nem meglepő, hogy reakció nélkül maradt Mezei Balázs azon kijelentése, hogy Gödel állítását isten létéről számítógéppel sikerült bebizonyítani (?). Nem tudom, Szalai miért nem csapott le legalább erre. (Megjegyzem, nagyon remélem, hogy ezt a kijelentést félreértettem.)
Én nem vagyok filozófus, így persze fennáll annak a lehetősége, hogy csak a képzetlenségem miatt éreztem súlytalannak és mondanivaló nélkülinek az egész beszélgetést. Bár, ha még én is hiányoltam a definíciókat és igazolásokat, valószínűleg jobb lett volna a közönség számára is, ha jobban összeszedett, logikai levezetések mentén haladó előadásokat hallhattunk volna.
A címben feltett két utolsó kérdés sem került elő. Pedig szerintem ebből a két szempontból éppen jól lehetett volna vizsgálni a "van-e isten?" felvetést is, legalábbis agnosztikus szemszögből.

Címkék: gonosz istenérv hitvita

> 81 komment

A bejegyzés trackback címe:

https://ateistaklub.blog.hu/api/trackback/id/tr765646349

Kommentek:

A hozzászólások a vonatkozó jogszabályok  értelmében felhasználói tartalomnak minősülnek, értük a szolgáltatás technikai  üzemeltetője semmilyen felelősséget nem vállal, azokat nem ellenőrzi. Kifogás esetén forduljon a blog szerkesztőjéhez. Részletek a  Felhasználási feltételekben és az adatvédelmi tájékoztatóban.

Wave43 2013.11.21. 10:04:34

"Szalai Miklós....emellett elmondta, hogy – bár isten és túlvilág nincs szerinte – valamilyen felsőbb teremtő létét nem lehet cáfolni."
Mivel több vallásban az isten a teremtő, v a teremtő az isten, ezért SzM nem sok mindent mondott: pontosabban annyit, h szerinte nincs, de nem lehet cáfolni, h van. Erre nem volt érdemes vitanapot rendezni...

Brendel Mátyás · http://ateistaklub.blog.hu/ 2013.11.21. 10:30:12

@Wave43: ahogy én ismerem Szalai álláspontját, ő a monoteizmus jóságos istenében nem hisz. ez a legfőbb kritikája. ahogy máshol írtam, a deizmust szerintem nem zárja ki. azt nem tudném megmondani, hogy egy személyes, de nem jóságos teremtőt kizár-e, vagy egy személytelen teremtő erőt tekint lehetőnek.
az sem világos, hogy ha nem cáfolható, akkor hisz-e benne, vagy nem.

én egyszer rendeztem egy estféleséget Szalaival, és én is ebbe ütköztem bele, hogy nem annyira és nem úgy ateista, mint a mainstream.

tudati 2013.11.21. 11:46:20

a vallásosok csak baromságot tudnak mondani, szóval igazából nem is lehet vitázni

pátyi 2013.11.21. 23:38:10

Szerintem Isten létét, vagy nem létét a tudomány eszközeivel nem lehet bizonyítani.Azért nem, mert a tudomány a teremtett világot vizsgálja, vizsgálhatja, Isten pedig nem része ennek a világnak. Isten egy külön világ, akivel, vagy amivel élő ember nem találkozik.
Nem igazán tudnám megmondani, hogy az ilyen vitának van-e értelme?
Az egyik ember hiszi Istent, a másik nem. A hitnek nincs köze ahhoz, hogy ki milyen okos, vagy inteligens.

Brendel Mátyás · http://ateistaklub.blog.hu/ 2013.11.22. 07:21:48

@pátyi: butaság olyan dologban hinni amelyet a tudomány. nem tud igazolni mert semmi közé a világhoz és haszontalan is

pátyi 2013.11.22. 15:53:16

@Brendel Mátyás:
A tudomány, a tudós alapvető, legfontosabb módszere, hogy felállít egy hihető elméletet, amit vagy tud bizonyítani, vagy nem. Attól, hogy nem tudja bizonyítani, még nem biztos, hogy butaság.
Az egyetlen tudomány a matematika, ahol ha egyszer valamit bizonyítottak, az örökre bizonyítva van.
Az összes többi tudományágra ez nem igaz. Mindíg hozzá lehet tenni, hogy jelenlegi tudásunk és meghatározott feltételek mellett igaz.
Szerintem Istenhit és tudományhit között nincs különbség.

tudati 2013.11.22. 16:14:35

@pátyi: nagyon nem mindegy hogy minek menyi a valószínűsége
például a holnapi napfelkeltének majdnem 100% a valószínűsége, míg a különleges hiper szuper isten létezésének kb. 0%
ha nagyon rendes akarok lenni akkor lehet mondjuk 50% is, de ha az eddigi bizonyítékokat nézzük, akkor valójában inkább a 0%-hoz van közel
habár még az ateisták közül is nagyon sokan nem értik miért fontos a valószínűség számítás

PHILO 2013.11.23. 10:30:55

Mezei Balázs Gödel kapcsán nyilván erre utalt:

www.heise.de/tp/artikel/39/39766/2.html

Sok egyéb helyen is olvasható erről, szaklapban is.

Wave43 2013.11.23. 21:51:14

@pátyi:
Bingo!!
Nagyon jól megfogalmaztad, pont ezt akartam írni. Lestél? :-))
A tudomany nem egyéb mint egy adott időpontban meglévő (azert nem teljes, azaz hiányos, azaz a nem teljes igazságot lefedő ) ismeretek v hiedelmek azon halmaza, mely egy adott premisszából indul ki. A tudomány hitéhez képest az istenhit v valláshit sokkal állandóbb. Nem beszélve arról h még a fizika különböző ágai v tételei (kvantumfizika vs relativitas elmélet) egymással összeegyeztethetetlen állításokat fogalmaznak meg. Tényleg, a matematikából kívül nincs "tudomány".

Wave43 2013.11.23. 22:15:10

@pátyi:
Arról nem beszélve, h a tudomany fetisizalva van, az nem tobb, mint a megismerés egy, azaz 1 útja, de semmi több, hiába nyomják egyes korlátolt elmék, hogy az kiemelt v felsőbbrendű e tekintetben Épp ezért ezek a korlátai is. Vallás vs tudomany viszonylatában lsd Gödel ontologiai istenérve

Koós István 2013.11.24. 12:35:19

@Wave43:

"A tudomany nem egyéb mint egy adott időpontban meglévő (azert nem teljes, azaz hiányos, azaz a nem teljes igazságot lefedő ) ismeretek v hiedelmek azon halmaza, mely egy adott premisszából indul ki."

A lófaszt hiedelem a tudomány. A tudomány modelleket alkot amelyeket igazol, vagy elvet. A modellek alapján preskripciók tehetők. A vallás erre nem képes, ez a különbség a kettő között.

"A tudomány hitéhez képest az istenhit v valláshit sokkal állandóbb."

1. Mint látod, a tudomány nem hit

2. Hol állandó neked az istenhit? Van politeizmus, monoteimzus, isten nélküli hit (buddhizmus), és a zsidó-keresztény vallás sem állandó, a biblia ellentmondás ellentmondás hátán.

"Arról nem beszélve, h a tudomany fetisizalva van, az nem tobb, mint a megismerés egy, azaz 1 útja"

Milyen más utat ismersz?

GERI87 2013.11.24. 13:23:42

@Wave43:

A tudomány a világról szerzett ismeretek összessége, ezek bővülnek és nem csak "cserélődnek" miközben egy helyben toporgunk, vagyis egyre többet tudunk...(szemben a vallásokkal amik csak annyit mondanak: "amit nem tudsz, nem értesz, az isten!")

és igen, vannak téves hipotézisek, na és?
Pont ez is mutatja hogy nem dogma, fejlődik, a tényekkel foglalkozik, a megismerhetővel, azzal ami hatással van ránk.

"A tudomány hitéhez képest az istenhit v valláshit sokkal állandóbb"

Mert nem tud semmi többet a világról mint amit a tudomány megállapít, vagy ami rosszabb, még azt is tagadja.
Állandóan hülyének maradni könnyű.

"....összeegyeztethetetlen állításokat fogalmaznak meg."

A vallások is ellentmondásban vannak egymással, de míg a tudomány észérveket használ és ezeket feloldja, addig a vallás üldöz, tagad, tilt.

"Arról nem beszélve, h a tudomany fetisizalva van, az nem tobb, mint a megismerés egy, azaz 1 útja,"

A megismerés egyetlen helyes és lehetséges útja.
Bárki kitalálhat egy vallást, istent...az pont annyira lesz hiteles és igaz mint a többi.
Tudomány viszont egy van, nincs olyan tudományos nézet ami szerint nincs gravitáció és nem érvényesek a fizika törvényei.
Ez lehetetlen, több istenben hinni viszont pont annyira lehet mint egyben.

Brendel Mátyás · http://ateistaklub.blog.hu/ 2013.11.24. 20:50:30

@pátyi: nagyon sok dolgot keversz, és kurva pontatlanul "gondolkodsz". vagy inkább az a helyesebb fogalmazás, hogy az a baj, hogy nem gondolkodsz el ezeken a kérdéseken rendesen.

"A tudomány, a tudós alapvető, legfontosabb módszere, hogy felállít egy hihető elméletet, amit vagy tud bizonyítani, vagy nem. Attól, hogy nem tudja bizonyítani, még nem biztos, hogy butaság."

1) a tudomány módszere nem a bizonyítás, hanem az igazolás
2) nem mindegy, hogy valamit a tudós nem tud igazolni, vagy a tudomány általában nem is tudhat igazolni. te is, és én is ez utóbbiról beszéltünk, oszt most meg jössz az előbbivel. szóval csúsztatsz rendesen.
3) nem azt írtam, hogy az a butaság, hogy a tudós nem tud valamit igazolni. a butaság az, ha hisz benne, holott nem igazolt. akár tudós, akár nem.

"Az egyetlen tudomány a matematika, ahol ha egyszer valamit bizonyítottak, az örökre bizonyítva van."

Az egyetlen tudomány a matematika, ahol egyáltalán bizonyításról beszélünk. A nem biztos "bizonyítás" nem bizonyítás, hanem csak igazolás.

"Az összes többi tudományágra ez nem igaz. Mindíg hozzá lehet tenni, hogy jelenlegi tudásunk és meghatározott feltételek mellett igaz."

igen.

"Szerintem Istenhit és tudományhit között nincs különbség. "

honnan a faszból húztad elő a tudományhitet?! mi a faszról beszélsz, ember?!

abból, hogy helyesen megállapítottad, hogy a tudomány a valós dolgokról sosem tudhatja biztosan, hogy mi az igazság, mindig hozzá lehet tenni, hogy "jelenlegi tudásunk mellett", ebből nem következik, hogy a tudomány elfogadott elméleteit igaznak elfogadni hit.

a hit az valami egészen más, az az, amikor a NEM IGAZOLT állítást fogadod el igaznak. ez a kurva nagy különbség tudás és hit között.

Miklós123 2013.11.25. 13:10:19

Tisztelt mindenki!
Az, hogy valaki mit minősít "hiábavalónak", "gyengének" mindig erősen szubjektív megítélés kérdése. Ámde sajnos a vita ismertetésébe becsúszott néhány kifejezetten sértő, illetve igazságtalan kitétel is, amelyeket itt kénytelen vagyok röviden kommentálni.
"Az ateizmus képviselője lehet, hogy azért szokott udvarias lenni, mert akkor legközelebb nem hívják meg, vagy gondol arra, hogy megütheti a bokáját.
***"
Nem azért vagyok udvarias minden, a vallásosakkal folytatott vitában, mert félek, hogy többször nem hívnak meg, vagy mert félek, hogy jön az Inkvizíció és máglyára küldenek, vagy ilyesmi. Azért vagyok udvarias, mert tisztelem az emberek vallásos érzéseit , szépnek tartom őket, sok szempontból hasznosnak (erről ott beszéltem is...), és becsülöm a vallásos gondolkodók szellemi teljesítményét.
Szerintem szerelmes emberekkel sem úgy kell vitatkozni, hogy azt mondom nekik: "Micsoda? A Klárika? Hát az egy hülye tyúk és el van hízva és iszik... és egyáltalán...". Hanem valahogy úgy, hogy "tisztelem az érzéseidet, lehet, hogy vannak a Klárikának bizonyos értékei, amik miatt rákattantál, de azért fontold meg, hogy...".
A vita hallgatója szememre veti, hogy miért nem csaptam le Mezei Gödellel kapcsolatos állításaira. Nagyon egyszerű: mert nem értek hozzá. Kurt Gödel nagyon nagy matematikus és formális logikus volt, megvizsgálta az ontológiai istenérvet és érvényesnek találta, amíg nem olvastam el és értettem meg a Gödel érvével kapcsolatos szakirodalmat, addig nem kritizálhatom.
A vita hallgatója eljuttatott hozzám egy fontos kérdést: "Hogyan hihet egy analitikus filozófus és ateista filozófus egy objektív moralitásban?" - amelyre ő nem kapott választ, ami őt rossz érzéssel töltötte el, úgy érezte, egy ilyen vitában erre válaszolni kellett volna.
Nagyon sajnálom. Erre a kérdésre, ebben a vitában valóban válaszolni kellett volna, egyszerűen azért, mert a teisták/ateisták közötti vitában az egyik döntő fontosságú kérdés az, hogy van-e objektív morál Isten nélkül.
Azért nem válaszoltam, mert nem jutott a kezembe a kérdés. Vagy a sok papír között, amelyekre a kérdéseket írták, elkeveredett, vagy valamiért átsiklott rajta a szemem, sajnálom.
Pótlólag a válaszom dióhéjban: azért hiszek ateista és analitikus filozófus létemre egy objektív moralitásban, mert úgy gondolom: igaz, hogy a morális vélekedések sokfélék (különböző társadalmakban és különböző kultúrákban mást-mást tartanak az emberek erkölcsösnek), de ez nem jelenti azt, hogy az erkölcsi vitákat ne lehetne észérvekkel eldönteni. Ugyanis a moralitás az emberi életnek egy bizonyos intézménye, az emberi életben tölt be egy bizonyos funkciót: szabályozza az emberek együttélését. Ésszerűen lehet amellett érvelni, hogy egyes történelmileg kialakult, s társadalmunkban elfogadott szabályok (például: "Tiszteld más tulajdonát!") alkalmasak ennek a funkciónak a betöltésére, hiszen nem élhetnek az emberek együtt akkor, ha nem tudják biztonságban a saját tulajdonukat, más szabályok viszont, amelyek szintén történelmileg kialakultak és bizonyos társadalmakban elfogadják őket (pl.: "nem házasodhatnak össze a melegek", "a feketék rabszolgának születtek" ) nem alkalmasak erre, hiszen az emberek nem élhetnek úgy együtt, hogy önkényesen elfogadott, kulturális hagyományokban gyökerező szabályok alapján egyes csoportjaikat a társadalom többsége diszkriminálhatja...
Természetesen az emberi együttélési szabályokról szóló viták nem mindig lezárhatóak, úgyszintén kérdés marad az, hogy hogyan lehet ezeket a szabályokat az élet konkrét helyzeteiben alkalmazni (pl. felülírhatja-e a "tiszteld más tulajdonát!" szabályt egy másik szabály: "ha lehetséges, magamat és másokat meg kell mentenem a haláltól, vagy a súlyos egészségkárosodástól!"). Ez azonban nem jelenti azt, hogy ez a vita reménytelen, hogy a moralitás ne volna objektív.

Brendel Mátyás · http://ateistaklub.blog.hu/ 2013.11.25. 14:15:42

@Miklós123:

Kedves Miklós,

"Szerintem szerelmes emberekkel sem úgy kell vitatkozni, hogy azt mondom nekik: "Micsoda? A Klárika? Hát az egy hülye tyúk és el van hízva és iszik... és egyáltalán...". Hanem valahogy úgy, hogy "tisztelem az érzéseidet, lehet, hogy vannak a Klárikának bizonyos értékei, amik miatt rákattantál, de azért fontold meg, hogy...". "

1) Szerintem egy szerelmes emberrel egyáltalán nem biztos, hogy kell vitatkozni. A szerelem ugyanis erősen szubjektív dolog. Ezzel szemben isten létezése nem szubjektív kérdés. Nagyon nehéz egy szerelmesnél egyáltalán olyasmit megfogni, hogy hibát követett el. Ha Klárika számomra ronda, de szerinte szép, akkor semmi értelme, hogy én vitázzak vele. Az, hogy ő szerelmes Klárikába, én nem, tökéletesen rendezi a helyzetet. Az már nyomósabb, ha mondjuk Klárika besurranó tolvaj, de még akkor is elég kérdéses, hogy bárkinek is azt a tanácsot adjam, hogy ne legyen szerlemes Klárikába, amikor az.

2) Az "udvarias" jelzőt mi nem annyira szó szerinti értelemben használtuk, inkább, hogy mennyire vitatkoztál erősen.

3) Mivel én nem voltam ott, ezért igazából én ezt meg se tudom ítélni, a beszámolóra hagyatkoztam. Ha a beszámoló nem említi meg egy csomó, kritikus érvedet, akkor egészen más a leányzó fekvése

4) A megjegyzésemben a "lehet" pontosan ezért szerepelt, illetve azért is, mert azt se tudom, mennyire jelenthet neked gondot szembe szállni a teológusokkal. Továbbá azért, mert én úgy sejtem, te nem azért nem vagy annyira militáns ateista, mert valamitől félsz, hanem valóban kevésbé vagy kritikus a vallással szemben, mint például én.

"mert nem értek hozzá. Kurt Gödel nagyon nagy matematikus és formális logikus volt, megvizsgálta az ontológiai istenérvet és érvényesnek találta, amíg nem olvastam el és értettem meg a Gödel érvével kapcsolatos szakirodalmat, addig nem kritizálhatom. "

Azon az alapon bárki kritizálhatja különösebb részletek ismerete nélkül, hogy nem lehetséges szintetikus apriori.

"Ésszerűen lehet amellett érvelni, hogy egyes történelmileg kialakult, s társadalmunkban elfogadott szabályok (például: "Tiszteld más tulajdonát!") alkalmasak ennek a funkciónak a betöltésére, hiszen nem élhetnek az emberek együtt akkor, ha nem tudják biztonságban a saját tulajdonukat, más szabályok viszont, amelyek szintén történelmileg kialakultak és bizonyos társadalmakban elfogadják őket"

Ez afféle naturalista morál, hiszen konszenzuális és praktikus. Szerintem a kérdező ezzel egyetértene, és ő objektív morál alatt abszolút morált értett, olyat, mint az isteni parancs morálját, amelyet akkor is követni kell, ha nem praktikus. De ezek szerint ilyenben nem hiszel.

Brendel Mátyás · http://ateistaklub.blog.hu/ 2013.11.25. 14:18:23

@Miklós123: töröltem a megjegyzésemet a bevezetőmben, mivel vitattad.

Koós István 2013.11.25. 15:54:40

Van másféle beszámoló is a vitáról, most tette ki Mezei Balázs az oldlára

www.atv.hu/kulfold/20131122-van-isten

PHILO 2013.11.25. 16:13:47

Én is ott voltam a vitán és nem osztom a cikkíró állításait. Szerintem kifejezetten jó vita volt. Először is úttörő, mert én nem tudok arról, hogy valaha volt is ilyen vita mifelénk. A kezdeményezés eleve üdvözlendő, s ezt kellett volna kihangsúlyozni. Az, hogy a vita nem volt elég éles, épp tán a tapasztalatlanságból ered. Miért várunk el szikrázó, svungos összecsapásokat, amikor még soha nem volt ilyen mifelénk? Tehát nagy pozítívum ez. A másik pozitívum vita baráti, megértő, fanatizmus-ellenes hangneme. Fanatikusok mindkét oldalon vannak, de láthatóan Szalay és Mezey nem tartoznak közéjük. Ezért nem is kellett, hogy nagyon összeveszekedjenek. Egyébként meg aki ismeri az ide tartozó problematika, irodalom legalább fontosabb elemeit, azt tudhatja, hogy mi mire vonatkozott, bár a felek nem tudták elég jól kifejteni. Ebben Szalay jobb volt, mert ő világosabban fogalmazott, de mezey eleve kontinentális vallásfilozófus, ezért másképp fogalmazott, amiben nekem sok minden tetszett. A legjobban az, hogy felhívta a figyelmet arra, a lét mást jelent különböző dolgok esetében, más a tárgyi és az alanyi lét, más a múltbeli és a jövőbeli, ezért az Isten léte sem egyszerűen megválaszolható. A vallással kritikusnak lenni, az könnyű. Azt a kérdést megválaszolni, hogy ha Dawkins szerint a vallás/religion egy pszeudojelenség, amely kontraszelekciós hatású, akkor hogy a fenében lehetséges, hogy az emberiség nemcsak az elmúlt évszázadig lényegében totál vallásos volt, hanem ráadásul eljutott a modernség ezer felfedezésére, a civilizációra? Stb. -- Szalay és Mezey kezet fogtak a végén és én azt szeretném, ha legközelebb moderátor nélkül ülnének le, aki nem épít pufferzónát közéjük.

Koós István 2013.11.25. 16:27:31

@Miklós123:

Néhány dologban szerintem nincs teljesen igaza.

az Istenről szóló könyvét én egyébként nagyon szeretem, nagyon alapos, éles logikájú, tömör és gondolatébresztő könyv, egyszer még gratuláltam is hozz a FB-on, meg írtam róla egy rövid posztot.

ateistaklub.blog.hu/2013/06/24/ha_letezik_isten_miert_nem_nyilvanvalo_a_lete

1. Viszont a szerelem és a vallás tisztelete közt vont páthuzam tényleg eléggé furcsa, ahogy Mátyás is írta. Ha a haverom azt mondja, hogy Gizike a legszebb, legtökéletesebb, sőt az egyetlen nő a világon, azt tiszteletben tartom, mert tisztában vagyok vele, hogy ez egy teljességgel szubjektív kijelentés, noha az objektív köntösben. Nem a a lány valóságos személyéről, a fiú érzelmeiről beszél - egyrészt, másrészt pedig ennek a megállapításnak nincs rám semmilyen következménye, hiszen a szerelem két ember magánügye (az egyetlen kivétel Don Quijote, aki mindenkivel el akarja ismertetni Dulcinea szépségét). Isten létérről mindez nem mondható el.

2. Az objektív morál is furcsa nekem. Mivel a morál közmegyezezésszerű, nem lehet objektív.

"Ésszerűen lehet amellett érvelni, hogy egyes történelmileg kialakult, s társadalmunkban elfogadott szabályok (például: "Tiszteld más tulajdonát!")"

A magántulajdon példája azért érdekes, mert ez is vitatott dolog, pl. Rousseau, nyomában Csokonay (!!), meg pl. a kommunisták pont a magántulajdont tartották erkölcstelennek, minden baj forrásának, úgyhogy nem is olyan objektív ez.

3. A vallás tisztelete sem egyértelmű.

"és becsülöm a vallásos gondolkodók szellemi teljesítményét."

Én nagyon szeretem pl. Szent Tamás teológiai summáját. Bizonyos szempontból teljesítmény, nekem tetszik az absztrakció magas szintje, a nyelvhasználat (tágabb értelemben véve) valóban nagyon invenciózus. De ettől még hülyeség. Az, hogy bizonyos szempontból teljesítménynek tartunk valamit (pl. Tamás esetében a nyelvi kreativitást), még nem zárja ki azt, hogy kerek perec kimondjuk róla, hogy bár költői alkotásnak szép, a valóságra vonatkoztatva hülyeség.

Koós István 2013.11.25. 16:40:57

@PHILO:

"épp tán a tapasztalatlanságból ered."

Elég komolyan képzett filozófusok mindketten, Mezeinek egy rakás könyvét láttam a múltkor a könyvtárban a vallásnál (200-as jelzet, ha jól emlékszem).

"A másik pozitívum vita baráti, megértő, fanatizmus-ellenes hangneme. Fanatikusok mindkét oldalon vannak, de láthatóan Szalay és Mezey nem tartoznak közéjük."

Ha egy vita éles és határozott, az nem jelent fanatizmut.

"A legjobban az, hogy felhívta a figyelmet arra, a lét mást jelent különböző dolgok esetében, más a tárgyi és az alanyi lét, más a múltbeli és a jövőbeli, ezért az Isten léte sem egyszerűen megválaszolható."

Na ez egy éktelen hülyeség, hogy Isten esetében mást jelent a lét, mint a dolgok esetében. Ez a vallási humbug.

"A vallással kritikusnak lenni, az könnyű."

Könnyű, nem könnyű, azok vagyunk.

Egyébként ha könnyű, akkor a vallásssal lehet valami baj, nem????

"Azt a kérdést megválaszolni, hogy ha Dawkins szerint a vallás/religion egy pszeudojelenség, amely kontraszelekciós hatású, akkor hogy a fenében lehetséges, hogy az emberiség nemcsak az elmúlt évszázadig lényegében totál vallásos volt"

Azon a kérdésen is gondolkodj el, hogy ha van vallás, akkor miért nem egyfajta vallás van, hanem ezerféle.

PHILO 2013.11.25. 17:04:07

@Koós István:

Kedves István! Én filozófus végzettségű vagyok és volt szerencsém Szalait és Mezeit is hallgatni (elnézést, rosszul írtam a nevüket.) Tatabányán tanítok és egyben doktorimra készülök hellenisztikus filozófia témában. Nem tudom, te hogy állsz a filozófiával, de amit írsz, abból nekem az jön le, hogy nem igazán. Az, hogy "Na ez egy éktelen hülyeség, hogy Isten esetében mást jelent a lét, mint a dolgok esetében. Ez a vallási humbug. - arra utal, nem ismered az irodalmat (Arisztotelész: "A létet sok értelemben mondjuk."), de a mai irodalmat sem, pl. Thomas Nagel Mind and Cosmos c. ez évi könyvét, ami nagy feltűnést keltett, a korábbi könyvei után ismét. Alaptétele ugye a szubjektivitás nem-objektivitása és az előbbi visszavezethetetlensége az utóbbira. S ezzel nincs egyedül, hogy ez "vallási humbug" lenne, az enyhén szólva is "éles és határozott". Amikor egy vita éles és határozott, az sokszor arra szolgál, hogy a kulturált beszélgetést beszólásokkal helyettesítsük, pl. "Ez egy vallási humbug." Kritikusnak lenni a tudománnyal szemben sokkal könnyebb. gondolj abba bele, hogy az emberiség ismert történetében a tudományos világkép legalább háromszor alapjában változott meg (Ptolemaiosz-kreáció-Newton-Einstein-Heisenberg). Aki ezek után azt mondja, hogy könnyű a tudománnyal szemben kritikusnak lenni, az nem téved nagyot. Utolsó megjegyzésedre nézve: miért lenne "ezerféle" vallás? Mit nevezünk vallásnak? A kannibalizmus, a cargo-kultusz nem vallások a szó valós értelmében, de még a mózesi hit sem vallás, ahogyan ezt az ortodox rabbik is állítják. Hogy lenne vallás? Parancsolat. Vallásnak egy összefüggő hagyományt nevezünk a kifejezés történetileg megalapozott értelmében, a többi legfeljebb, ahogyan mondani szokás, Religionsähnliche Erscheinungen.

Egyébként most vettem észre, hogy a cikket Brendel M. írta, aki megvallja, hogy nem is volt ott a vitán. Hogy lehet ekkor lelkiismeretesen írni a vitáról? Vagy miféle hozzáállás ez?

Moralitás: miért zárná ki egymást az objektivitás és a konszenzus? A tudósok között va konszenzus van arról, hogy az Intelligent Design nagyrészt butaság, de ettől ez még ostobaság, objektíve. Nem?

Brendel Mátyás · http://ateistaklub.blog.hu/ 2013.11.25. 17:13:40

@Koós István: ez részletesebb beszámoló, sok részlet van benne, és elhangzanak jó érvek is.

azért Mezeinek azt elengedni, hogy ha az emberiségnek csak mondjuk 10-20 százaléka ateista, akkor isten nem is rejtőzködik, az magas labda volt.

mert:

1) az a 10-20 százalék se semmi
2) abban a 10-20 százalékban vana tudósok nagy része
3) továbbá az emberiség további mondjuk 60 százaléka legalábbis rossz istenben hisz. erre se mondhatjuk, hogy isten nem rejtőzködik

az írástudásos dologra is roppant frappáns érv lett volna, hogy a tudósok nagy része ateista. tehát pont az irástudóbbak. azért az iszonyat irreális feltételezés, hogy pont az intelligensebb, tanultabb emberek ne tudnának a "csillagokból olvasni", hogy nekik kéne megtanulni valamit, nem pedig Marcsi néninek Hajdúborzasztóról.

Brendel Mátyás · http://ateistaklub.blog.hu/ 2013.11.25. 17:22:40

@PHILO:

", a lét mást jelent különböző dolgok esetében, más a tárgyi és az alanyi lét, más a múltbeli és a jövőbeli, ezért az Isten léte sem egyszerűen megválaszolható."

ez ugyebár elég nagy vakítás. a lét bizony ugyanazt jelenti minden valóságosnak tekintett létezőnél. a múlt, jelen és jövő ebben egy kis különbséget jelent, de ennyivel nem lehet elmismákolni a dolgot.

és pláne azzal nem lehet, ha valaki elkezd egy ilyen nagyívű mismákolást, és a végén nem tisztázza azt, hogy akkor mit is jelent az, hogy "létezik isten", van-e rá igazolás, és ha van, miért nincs.

"megválaszolni, hogy ha Dawkins szerint a vallás/religion egy pszeudojelenség, amely kontraszelekciós hatású"

nem biztos, hogy Dawkins szerint kontraszelekciós hatású.

"akkor hogy a fenében lehetséges, hogy az emberiség nemcsak az elmúlt évszázadig lényegében totál vallásos volt"

de ha kontraszelekciós hatású, akkor Dawkins erre ugye azt mondja, hogy a vallás egy káros mém. azaz a vallás bár előnytelen a homo sapiensnek, de mint mém élősködik rajta. ahogy sok más élősküdőt is ismerünk. az influenzavírus ugyebár szintén kontraszelekciós hatású a homo sapiensnek, de az influenzavírus ettől ugye még frankón megvan, és nem jelent problémát a léte evolúciós szempontból.

PHILO 2013.11.25. 17:24:15

@Brendel Mátyás: Kedves Mátyás! Nem ismered a felmérést, láthatóan. Mezei szerint ez a Gallup 2000 Millianiumi Felmérés, amelyet levettek a netről, de nyomtatottan hozzáférhető. Emlékeim szerint olyan kérdés is van, hogy kiféle az isten, s amiben az emberiség nagy része hisz, az egyetemes szellem, mindenható ágens, ilyesmi. Tehát nem "rossz isten", de egy általános teizmus. -- Továbbá: Az elmúlt 200 év vezető tudósai döntő részben teisták. Einstein, Heisenberg és Gdel is ide tartoznak. Ettől még igaz lehet, hogy manapság a csúcskutatók többsége inkább ateista (erről Dawkins beszél), bár én ilyen felmérést nem láttam. Végül nem 10-20, hanem 10% alatti az ateisták száma. S ez nagy különbség!

PHILO 2013.11.25. 17:30:43

@Brendel Mátyás: Nem mismásoltam el semmit, amikor azt mondtam, hogy a lét a mai csúcstudomány szerint sem egyértelmű. A szubjektivitás nem objektivitás, a múlt nem jelen vagy jövő. A gondolati lét nem fizikai lét. A képzelet nem realitás. Ezek kategorikus különbségek. Tehát a létet sokféleképpen mondjuk. Mindezt redukálni egy kezdetleges (de soha nem is létezett) empirizmusra -- szuper. A kvantumelmélet ráadásul úgy borít minden primitív empirizmust, mint nagypapa a tele bilit éjszaka. A Van Isten? Kérdés nemcsak arra jó, hogy kiderítsük, igazolható-e, hanem arra is, hogy felfogjuk, a létet sokféleképpen mondjuk (Arisztotelész). S arra is, hogy megvizsgáljuk, mit jelent az igazolás. Gödel érvének számítógépes igazolását elvégezte Benzmüller, s megadtam az ide tartozó linket fentebb. Miért nem olvasod el? Miért nem válaszolsz rá? Miért nem cáfolod? -- Dawkins túl sokat markol és keveset fog. Az influenza nem halálos, a vallás szerinte igen, ennek ellenére az ismert emberiségtörténet 90% tele van vallással vagy valláshoz hasonló formákkal, de többet mondok: a modern tudomány valamennyi nagy alakja istenhívő volt. Biztosan lebetegedve és halálosan megfertőzve, de Istenben híve hozták létre a modern tudomány csúcsteljesítményeit.

Brendel Mátyás · http://ateistaklub.blog.hu/ 2013.11.25. 17:33:34

@PHILO:

"pl. Thomas Nagel Mind and Cosmos c. ez évi könyvét, ami nagy feltűnést keltett, a korábbi könyvei után ismét. Alaptétele ugye a szubjektivitás nem-objektivitása és az előbbi visszavezethetetlensége az utóbbira."

Nagel önmaga is egy iszonyat zavaros filozófus. jó helyen kapisgál a szubjektívvel és objektívvel, csak azt nem értem, miért gabalyodik bele. pedig egyszerű: a szubjektív kérdések mások, mint az objektívek. a szubjektív kérdésekre nincs igaz vagy hamis válasz. a szubjektív kérdések az értelmes lények relációit reprezentálja. tehát VISSZAVEZETHETŐEK AZ OBJEKTÍVRE, és ezzel el is van intézve a dolog.

ha Thomas úgy találja, hogy "Julcsi szép" nem egy misztikum, hanem egy reláció. és nem is a létezés másik formája. Julcsi szépsége nem egy külön létező, nem egy másképp létező dolog, hanem egy esztítikai adat Thomas agyában. ennyi.

" Kritikusnak lenni a tudománnyal szemben sokkal könnyebb. gondolj abba bele, hogy az emberiség ismert történetében a tudományos világkép legalább háromszor alapjában változott meg (Ptolemaiosz-kreáció-Newton-Einstein-Heisenberg). Aki ezek után azt mondja, hogy könnyű a tudománnyal szemben kritikusnak lenni, az nem téved nagyot."

te filozófián érveléselméleten azt tanultad, hogy ez érv?!

" Utolsó megjegyzésedre nézve: miért lenne "ezerféle" vallás? Mit nevezünk vallásnak? A kannibalizmus, a cargo-kultusz nem vallások a szó valós értelmében, de még a mózesi hit sem vallás, ahogyan ezt az ortodox rabbik is állítják. Hogy lenne vallás? Parancsolat. Vallásnak egy összefüggő hagyományt nevezünk a kifejezés történetileg megalapozott értelmében, a többi legfeljebb, ahogyan mondani szokás, Religionsähnliche Erscheinungen. "

azért ha a zsidó vallást sem nevezed vallásnak, úgy könnyú kimosni a saját kis szektádat, hogy egyedülálló. csak mit szólnak ehhez a zsidók?! és miért volna neked ebben igazad?! és aztán mit kezdesz a buddhizmussal, hindu vallással, pogány vallások tömkelegével, az iszlámmal, a bahai vallással, a mormonizmussal, és a többivel?! ezek mind nem vallások, csak a te kis szektád a vallás?! ez azért elég olcsó, átlátszó, és soviniszta duma.

"Hogy lehet ekkor lelkiismeretesen írni a vitáról?"

nem írtam a vitáról.

"Moralitás: miért zárná ki egymást az objektivitás és a konszenzus?"

nem zárja ki, de a konszenzust akkor szokás emlegetni, ha valami csak konszenzus. itt most erről volt szó.

Koós István 2013.11.25. 17:36:44

@PHILO:

"Egyébként most vettem észre, hogy a cikket Brendel M. írta, aki megvallja, hogy nem is volt ott a vitán. Hogy lehet ekkor lelkiismeretesen írni a vitáról? Vagy miféle hozzáállás ez?"

A cikket nem Mátyás írta, hanem egy kommentet illesztett be változtatás nélkül, ezzel kezdődik a szöveg, olvasd el.

"de a mai irodalmat sem, pl. Thomas Nagel Mind and Cosmos c. ez évi könyvét"

Ezt valóban nem ismerem. Én a keresztény ontológiára (létanalitika, vagy mi) gondoltam, ami szerint a létezésnek fokozatai vannak, pl. Bolberitz könyvében, vagy Szent Tamásnál, aki szerint Isten a lét tiszta aktualitása, tiszta létaktus (ami az általad hivatkozott Arisztotelésztől eredő aktus-potencia tan átírása). Ebben az értelemben mondják a keresztény gondolkodók, hogy Isten léte nem olyan, mint a más dolgok léte; Isten tovább transzcendens és immanens stb. Erre gondoltam, amikor humbugnak neveztem, mert ez nem egyéb, mint játék a szavakkal.

"Amikor egy vita éles és határozott, az sokszor arra szolgál, hogy a kulturált beszélgetést beszólásokkal helyettesítsük, pl. "Ez egy vallási humbug."

A beszólás nem vita. Sem éles, sem életlen, nem vita.

"Kritikusnak lenni a tudománnyal szemben sokkal könnyebb. gondolj abba bele, hogy az emberiség ismert történetében a tudományos világkép legalább háromszor alapjában változott meg (Ptolemaiosz-kreáció-Newton-Einstein-Heisenberg)."

Ehhez aztán végképp nem értek, de amennyire tudom, az einsteini fizika nem váltotta le a newtonit, ez utóbbi kisebb méretekben nagyon is jól működik. Úgyhogy nincs arról szó, amivel te érvelni akarsz, hogy a tudomány sem megbízható, mert az is mindig változik.

"Utolsó megjegyzésedre nézve: miért lenne "ezerféle" vallás?

Figyelj, van egyistenhit, annak egy rakás változata, (a keresztény Isten meg a gonosz aljas őszövetségi Jahve pl. eléggé különböző istenek), van a politeizmus Zeusszal, Amon-Rével meg még másokkal, meg van Isten nélküli vallás is, a buddhizmus. Ez azért eléggé sokféle, nem?

Brendel Mátyás · http://ateistaklub.blog.hu/ 2013.11.25. 17:37:11

@PHILO:

"Mezei szerint ez a Gallup 2000 Millianiumi Felmérés, amelyet levettek a netről, de nyomtatottan hozzáférhető."

én ennél újabb Gallup felmérést ismerek.

"Továbbá: Az elmúlt 200 év vezető tudósai döntő részben teisták."

viszont a jelen tudósai döntő részben ateisták. ez pedig:

1) több tudóst jelent, ugyenis a tudósok nagy része ma él
2) a legtájékozottabb és legintelligensebb tudósokat jelenti. up to date tudással

nehéz lesz kimagyaráznod.

"10% alatti az ateisták száma. S ez nagy különbség!"

a WP szerint 11 százalék

en.wikipedia.org/wiki/Major_religious_groups

ami roppant kínos nektek.

Koós István 2013.11.25. 17:45:12

@PHILO:

"A gondolati lét nem fizikai lét. A képzelet nem realitás."

Jó, akkor ezek szerint eddig van kétféle lét: valóságos, reális, meg képzeletbeli.

Isten melyikbe tartozik? Ha a reális, fizikai létbe, akkor ugyanolyan a léte, mint az objektumoké, ha pedig a képzeletbeli létbe, akkor azzal elismered, hogy nem létezik a valóságban.

"A kvantumelmélet ráadásul úgy borít minden primitív empirizmust, mint nagypapa a tele bilit éjszaka."

Ezt meg honnan vetted? a kvantumos világ másképpen viselkedik, mint a nagyobb méretű világ, de ez nem jelenti azt, hogy nem lenne empirikus világ. Megvannak a szabályszerűségei, kutatható, leírható.

Brendel Mátyás · http://ateistaklub.blog.hu/ 2013.11.25. 17:45:18

@PHILO:

"Nem mismásoltam el semmit, amikor azt mondtam, hogy a lét a mai csúcstudomány szerint sem egyértelmű."

de.

"A gondolati lét nem fizikai lét."

így van. no de ha isten csak a gondolatban létezik, fizikailag nem, azt nevezzük mesének. akkor isten olyan, mint hófehérke, a hétfejú sárkány, és az aranyhegy.

" A képzelet nem realitás. Ezek kategorikus különbségek."

így van. a képzeletbeli, fiktív dolgok azok nem a lét egy másik formái, hanem NEM LÉTEZNEK. legalábbis azok így mondják, akik egyenesen beszélnek.

"A kvantumelmélet ráadásul úgy borít minden primitív empirizmust, mint nagypapa a tele bilit éjszaka."

a primitívet lehet, de hát te itt nem primitív empiristával beszélsz, kisfiam.

"Gödel érvének számítógépes igazolását elvégezte Benzmüller, s megadtam az ide tartozó linket fentebb."

1) HAZUDSZ, nem adtad meg.

2) ismerem, baromság. nem lehetséges szintetikus a priori. Gödel bizonyításán egyébként kurvára semmit sem kell számítógépesen igazolni, az egész médiahumbug.

Szalaival ellentétben ÉN ÉRTEK HOZZÁ, nekem nem adod be.

" Miért nem olvasod el? Miért nem válaszolsz rá? Miért nem cáfolod?"

válaszoltam rá:

ateistaklub.blog.hu/2013/10/30/lehet-e_fotelbol_macbookkal_bizonyitani_isten_letet

inkelve van a postból, te barom. miért nem nézel körbe, mielőtt ugrálsz?!

"Az influenza nem halálos, a vallás szerinte igen"

az influenza és a vallás is pont annyira halálos, hogy még tudjon terjedni.

"az ismert emberiségtörténet 90% tele van vallással vagy valláshoz hasonló formákkal"

influenzával is.

"a modern tudomány valamennyi nagy alakja istenhívő volt."

a kortárs tudomány valamennyi nagy alakja ateista.

Brendel Mátyás · http://ateistaklub.blog.hu/ 2013.11.25. 17:53:35

@PHILO:

a következőt gondold meg! Lehet, hogy régen a tudósok hívők voltak, de ma a tudósok nagy része ateista. Lehet, hogy ma az emberek nagy része hívő, de a tudósok nagy része ateista. Továbbá az emberek nagy része úgy hívő, hogy a tanult, tájékozott, intelligensebb, normális életet élő emberek a fejlettebb országokban jelentős részben, akár többségében is ateisták.

Ezt hogy magyarázod ki?! Mert nekem akárhogyis de ebből az jün le, hogy a hit a tudatlanságból és nyomorúságból táplálkozik, és a nép ópiuma.

PHILO 2013.11.25. 18:25:58

Több megjegyzésre válasz (de nem tudok mindenre válaszolni):

Kedves Mátyás!

Véleményem szerint te nem ismered Nagelt. Vagy ha olvastad is, kellő figyelem nélkül. Nagel írásai mindenki számára elérhetőek, stílusuk egyszerű és világos, az „iszonyat zavaros” (magyarul: „iszonyúan zavaros”) nem jellemző rá. Közismert, tisztelt, és noha sokan vitáznak vele, azt kevesen tagadják, hogy nagyon fontos felfedezést tett (egyébként T. Sprigge nyomán) a „what is it like to be…?” kérdésének a felvetésével. Te persze tagadhatod ennek fontosságát, de meg kellene mondanod, hogy milyen vonalon: az eliminatív fizikaizmus vonalán? A funkcionális fizikalizmusén? Az epifenomenalizmus értelmében? Vagy egy további elgondolásod van?

A továűbbi bekezdés mutatja, hogy nem érted, vagy félreérted a problémát. Nem a köznyelvileg szubjektívnek nevezett relációkról van szó (emotivizmus), hanem magáról a szubjektívitásról mint olyanról. Kérlek, nézz utána ennek a problémának.

Én nem kívántam egy formalizált érvet alkotni a történetileg változó tudományos minták létéből. De ha ez a kívánságod, valamikor meg is tehetem. A helyzet a következő: alapvetően szembetűnő, hogy a tudományos minták fél évszázadonként megváltoznak. Erre nem én hívtam fel a figyelmet, hanem olyan jelentékeny tudománytörténész, mint Th. Kuhn. Nagyon is elgondolkodtató, hogy voltaképpen mi történik egy tudományos minta megváltozásakor. S az a kérdés teljes mértékben legitim, hogy vajon a mai standard tudományos felfogást nem fogja egy néhány évtized vagy évszázad után egy élesen eltérő felfogás felváltani. Gödel nem teljességi tétele nagyon jó példa erre, mivel ez a matematikusok ezredéves teljességi törekvését egyszer s mindenkorra lezárta. Általában úgy szokták ezt értelmezni, mint ami egy tudományos világkép teljességére is vonatkoztatható.

Kérlek, nézz utána annak, hogy az ortodox, hithű zsidóság vallásnak tartja-e a maga hagyományát. Nem tartja annak. Isten kinyilatkoztatása (gala) a zsidóságnak lett adva a parancsolatok formájában. Ez nem „religio”, hanem követés, hit, megtartás, szabály. Ez nem vitatéma. Természetesen vannak népszerű alkotók, akik a zsidóságot is „világvallásnak” nevezik (Glasenapp, Sart stb.), de ez az ő értelmezésük, nem a zsidóságé. Hasonlókat mondhatunk e a buddhizmusról, a hinduizmusról stb. Mindezt a nyugati nyelvek hosszú időn át nem vallásnak, hanem kultusznak nevezték (nem religiónak, hanem cultus-nak). Az iszlám volt az első, amelyre a nyugati szóhasználat kiterjesztette a „religio” elnevezést, de ez is csak a 18. századtól terjed el. Addig az iszlám „secta” (= tanítványcsoport). – Kár, hogy ezt itt el kell mondanom, mert Mezei száz és száz oldalt íRt ezekről a kérdésekről, s én is az ő könyveiből tudom, amelyek megbízhatóan jegyzeteltek, hivatkozottak.

A moralitás nem lehet CSAK konszenzus dolga. Erre statisztikai bizonyítékkal szolgálhatunk, mivel az emberi élet minden emberi kultúrában lényegi védelem alatt áll (ami nem zárja ki adott esetben a feláldozását, de a gyilkosságot igen).

Ami a tudósok statisztikáját illeti: én még nem láttam mai statisztikát a vezető tudósokról. Csak állításokat hallottam. Az viszont tényszerűen igazolható, hogy a jelentős tudósok, akik tehát korszakalkotót értek el, nagyrészt istenhívők voltak. – Hogy a mai emberiség 10% ateista, én kétlem, de azt nem kétlem, hogy vannak jelentős számban ateisták. Viszont a vita során az hangzott el, hogy ha isten tényleg rejtette, akkor miért hiszik annyian, hogy létezik? Az „annyianhoz” elég lenne, ha a mai emberiség fele hinné ezt. De nem a fele hiszi, hanem sokkal több.

A kvatumelméletre vonatkozó nézeteimet több forrásból merítem, mások melett L’Espagnat-tól és Stephen M. Barr.tól. Mindkettő egybehangzó állítása szerint nincs olyan vallási „fantazmagória”, ami túlhaladna azokon az elképesztő következményeken, amelyek a kvantumelmélet széles körű áttekintéséből erednek. Ilyen egyebek mellett D’Espagnat (csúcskutató) azon állítása, amely szerint a kvantumelméletből egyenesen következik egy világfenntartó elme/szellem létezése. Nekem ez meggyőzőnek látszik. De említheném Spaemann istenérvét is, amely szerint a futurum exactum grammatrikai léte, s az ide vonatkozó ontológiai következmény, posztulálja (= szigorúan megköveteli) Istennek mint egyetemes szellemi ágensnek a létét (ezt most nincs hely kifejteni, de elolvasható).

Sajnos nem tudok a továbbiakra reagálni, ezért összefoglalóul csak annyit: Isten léte v. sz. igazolható, bizonyítható, de ezzel csak egy valószínűséghez jutunk, ami messze áll a hívő meggyőződésétől. De e meggyőződés alapossága mellett is jól lehet érvelni.

Egyébként lebaromozni a másik felet, az nem menő. Megnéztem, mit válaszoltál, s az szerintem nem válasz.

PHILO 2013.11.25. 18:34:27

Idemásolom Benzmüller válaszát, amely megmutatja egy értelmes kritika lehetőségét, egyben válaszol is erre a lehetőségre, körültekintően (s ezt a linket adtam meg fentebb is).

Ezzel az idézettel bezárom itteni tevékenységemet. Néhány megjegyzésemre a következő kitételek jöttek: faszság, barom, hazudsz. Ez egy igazi ateista klub --! Bye.

"Christoph Benzmüller: Es gibt sehr unterschiedliche Möglichkeiten, sich der Frage nach Gottes Existenz zu nähern. Häufig wird sie als eine reine Frage des Glaubens aufgefasst, und diese Sicht steht einer abstrakten, logik-zentrierten Herangehensweise, wie von Gödel praktiziert, entgegen. Zunächst steht also die übergeordnete Frage im Raum, welche Herangehensweise an das Thema jeder Einzelne für sich akzeptiert und welche er grundsätzlich ablehnt. Die Reduzierung der Frage auf begrifflich-logische Untersuchungen wird von nicht wenigen als eine inakzeptable Extremform angesehen. Diese Form der Auseinandersetzung ist es aber, die das Logikgenie Kurt Gödel gewählt hat.

Selbst wenn wir die grundsätzliche Herangehensweise von Gödel akzeptieren, so gibt es dennoch Aspekte, die wir kritisch hinterfragen müssen. Eine zentrale Frage ist natürlich, ob wir Gödels Grundannahmen (d.h. seine abstrakten Begriffsdefinitionen und seine Axiome) sowie deren konkrete Kodierungen in dem zugrunde gelegten Logikformalismus akzeptieren. Eine dazu orthogonale Frage ist, ob der Logikformalismus adäquat gewählt ist. Akzeptieren wir aber all diese Punkte (Herangehensweise, Logikformalismus und Grundannahmen), so sollte man sich wohl auch mit der Konsequenz, der notwendigen Existenz Gottes, auseinandersetzten.

Ich muss zugeben, dass ich all diese Punkte für mich selbst noch nicht erschöpfend beantwortet habe, und möglicherweise war dies auch bei Gödel der Fall. Wie dem auch sei, unsere aktuellen Arbeiten eröffnen interessante neue Möglichkeiten, den Logikformalismus und die Grundannahmen als veränderbare Parameter aufzufassen und dann mit diesen Parametern im Theorembeweiser zu experimentieren. Dadurch sollte es uns möglich sein, die Stichhaltigkeit weiterer Gottesbeweise zu untersuchen und diese Gottesbeweise zu variieren, um möglicherweise neue Einsichten zu gewinnen. Man kann also sagen, dass wir interessante neue Perspektiven für eine Computer-assistierte theoretische Philosophie bzw. Metaphysik aufzeigen. Nicht mehr, aber auch nicht weniger."

Koós István 2013.11.25. 18:58:41

@PHILO:

Megsértődni lehet, de olvad vissza, ami írtál. Figyelj, te nem mondasz semmit, csak kioktatsz.

először is közlöd, hogy Mátyás nem olvasta Nagelt, meg nem érti. Ebben az esetben a tisztességes az lenne, ha megmondanád, mit nem ért, vagy mi az, amit másként kell érteni. Mert így nekem az a benyomásom, hogy te itt egy olyan szerzőt dobtál be, akit te sem ismersz. blöfföltél.

Aztán a vallásokról beszélsz. közlöd, hogy a zsidó hit nem vallás. Hogy miért nem, hogy mi a vallás, és ma a nem vallás, mi a kultusz a te szavaiddal, ezt nem mondod meg.

I"sten kinyilatkoztatása (gala) a zsidóságnak lett adva a parancsolatok formájában."

A keresztény hit is kinyilatkoztásról beszél.

"A kvatumelméletre vonatkozó nézeteimet több forrásból merítem, mások melett L’Espagnat-tól és Stephen M. Barr.tól. Mindkettő egybehangzó állítása szerint nincs olyan vallási „fantazmagória”, ami túlhaladna azokon az elképesztő következményeken, amelyek a kvantumelmélet széles körű áttekintéséből erednek."

Ez puszta értetlenkedés. A kvantumelmélet csak neked elképesztő, attól még empirikusan vizsgálható, tudományosan leírható.

"Ilyen egyebek mellett D’Espagnat (csúcskutató) azon állítása, amely szerint a kvantumelméletből egyenesen következik egy világfenntartó elme/szellem létezése."

Na, ez már tényleg a hülyeség. Ezt te komolyan gondolod???!!!!

Magyarázd már el, hogyan következik a kvantumelméletből Isten léte!!!!

Én nem használtam csúnya szavakat, úgyhogy nyugodtan válaszolhatsz. De nem azzal, hogy "nézz után", mert ez nem érv, ez nem vita, ez a tudatlanság megnyilatkozása.

Koós István 2013.11.25. 19:02:33

@PHILO:

Egyébként ez jellemző a vallásos kommentelőkre. Előhozakodnak azzal, hogy miket kellene elolvasni, de azt már nem tudják megmondani, hogy mi van abban a könyveb, amit ajánlanak. Könyvet ajánlani mindenki tud, és pl. mindjárt belinkelek a Széchenyi könyvtár honlapját, hogy olvasd el azt.

PHILO 2013.11.25. 19:58:57

@Koós István: A kvantumelmélet szerint egy adott elektron adott helyen és adott időben meghatározhatatlanul vagy bizonytalanul részecske vagy hullám természetű. A kérdés az, hogy ez bizonytalanság-e (uncertainty) vagy meghatározatlanság (indeterminacy). D’Espagnat sokakkal együtt úgy látja, hogy ez meghatározatlanság, amiből több következtetést szűr le: 1. A lokális realitás által felmutatott nagy objektivitás észlelésünk eredménye. 2. A realitás, ahogyan az a kvantumkísérletekben feltárul, egy mélyebb valóságra utal, amely lehetővé teszi a megfigyelő és a megfigyelt összefüggését. 3. Ezt a mélyebb valóságot „leplezett realitásnak” nevezi (veiled reality), amely a tudomány számára csak és kizárólag a kísérletek kapcsán fejeződik ki, ami olyan, mint amikor éjszaka egy zseblámpával belevilágítunk a sötét erdőbe: világosan látunk mindent, ami a fénycsóvába belefér, de nem tudjuk, mi van kívüle. 4. Ez a mélyvalóságot mégis felfogjuk, mert ez minden tapasztalat, kísérlet, gondolkodás alapja, a kvantumkísérletek és egységes leírásuk alapja. 5. Ezt a mélyvalóságot érzelmekben, esztétikai élményekben, vallási tapasztalatban sejtjük meg, de nem fogalmilag. 6. Ezt a mélyvalóságot Istennek nevezhetjük. 7. Leszögezhető, hogy minden valóság – a lokális realitás tapasztalatától eltérően – csak mint megfigyelt valóság állhat fenn, mivel a megfigyelő és a megfigyelt elválaszthatatlansága a kvantumelmélet egyik alapbelátása. 8. Mivel az ember nem létezett mindig, sőt a földi élet sem, sőt semmiféle élet sem létezett vagy fog létezni mindig, fel kell tennünk, hogy van „végső megfigyelő”, aki figyelemmel kísérte a valóság létrejöttét és akkor is „megfigyeli”, amikor biológiai alapú értelmes lények nem léteznek majd. Azaz fel kell tételeznünk, hogy létezik egy olyan szuper-lény, aki közel áll a vallásban hirdetett istenhez.

Brendel Mátyás · http://ateistaklub.blog.hu/ 2013.11.25. 19:59:28

"nagyon fontos felfedezést tett (egyébként T. Sprigge nyomán) a „what is it like to be…?” kérdésének a felvetésével."

szerintem egy nagyon banális zavart okozott, s ahogy az már lenni szokott, ha ismert filozófus mond valami butaságot, akkor a filozófia egy ideig elcsócsálja. aztán történelem lesz belőle. és száz év múlva csak mosolyognak rajta.

egyébként ez ugyebár egy terelés részedről.

" Te persze tagadhatod ennek fontosságát, de meg kellene mondanod, hogy milyen vonalon: az eliminatív fizikaizmus vonalán? A funkcionális fizikalizmusén? Az epifenomenalizmus értelmében? Vagy egy további elgondolásod van?"

eliminatív nyelvfilozófia. a kérdés értelmetlen. szubjektív-objektív kategóriasértés. a "milyen denevérnek lenni?" nem egy értelmes teoretikus kérdés. a "milyen Brendel Mátyásnak lenni" kérdésre sem tudok válaszolni, és nem azért, mert ez valami misztikus dolog, hanem "amiről nem lehet beszélni, arról hallgatni kell".

"Nem a köznyelvileg szubjektívnek nevezett relációkról van szó (emotivizmus), hanem magáról a szubjektívitásról mint olyanról."

ez uge ugyanaz a szófordulat, hogy "nem a piros dolgokról van szó, hanem a pirosságról", vagy "nem a létező dolgokról van szó, hanem a Daseinről". a metafizikusok egyik gyakori mintája arra, hogyan hozzanak létre értelmesnek látszó, de megtévesztő, értelmetlen kérdéseket.

ha nincs szó konkrétan az emberek szubjektív dolgairól, akkor sajnálom, de nem fogadom el a kérdésfeltevést.

"Én nem kívántam egy formalizált érvet alkotni a történetileg változó tudományos minták létéből."

tudom, te csak retorikai fogásként kívántad alkalmazni, de ha felsülsz vele, behúzod a farkad.

" Erre nem én hívtam fel a figyelmet, hanem olyan jelentékeny tudománytörténész, mint Th. Kuhn."

ugyan már, kicsihuszár, nem Kuhn találta fel, hogy a tudományos elméleteket néha megdöntik vagy elvetik.

" Nagyon is elgondolkodtató, hogy voltaképpen mi történik egy tudományos minta megváltozásakor. S az a kérdés teljes mértékben legitim, hogy vajon a mai standard tudományos felfogást nem fogja egy néhány évtized vagy évszázad után egy élesen eltérő felfogás felváltani."

igen, csak vegyük észre. még mindig terelsz, és még mindig sehol nem vagy ahhoz, hogy legyen egy érved. sokat fecsegsz, de nincs érved. puffogtatod a filozófiai közhelyeket.

" Gödel nem teljességi tétele nagyon jó példa erre, mivel ez a matematikusok ezredéves teljességi törekvését egyszer s mindenkorra lezárta."

de nem vonatkozik az empirikus tudományokra.

math.uni.hu/angol/Carnap-Popper-Godel-MFSZ.pdf

Brendel Mátyás · http://ateistaklub.blog.hu/ 2013.11.25. 20:00:58

@PHILO:

"Kérlek, nézz utána annak, hogy az ortodox, hithű zsidóság vallásnak tartja-e a maga hagyományát. Nem tartja annak."

"Judaism (from the Latin Iudaismus, derived from the Greek Ἰουδαϊσμός, and ultimately from the Hebrew יהודה, Yehudah, "Judah";[1][2] in Hebrew: יהדות, Yahadut, the distinctive characteristics of the Judean ethnos)[3] is the religion, philosophy and way of life of the Jewish people."

en.wikipedia.org/wiki/Judaism

"היהדות כדת היא הראשונה מבין שלוש הדתות המונותאיסטיות המערביות."

he.wikipedia.org/wiki/%D7%99%D7%94%D7%93%D7%95%D7%AA

ez a többségi zsidó vélemény. a kisebbségi vélemény irreleváns.

"A moralitás nem lehet CSAK konszenzus dolga. Erre statisztikai bizonyítékkal szolgálhatunk"

ennek az állításnak már a logikáját se értem. mi a halált akarsz te itt mondani?!

"mivel az emberi élet minden emberi kultúrában lényegi védelem alatt áll"

hivatkozás?! egyébként ha ez igaz, ez csak azt mutatja, hogy vannak az egész emberiségre jellemző konszenzusok. egyébként az csak a modernebb kultúrákra jellemző.

"Ami a tudósok statisztikáját illeti: én még nem láttam mai statisztikát a vezető tudósokról."

ha tudatlan vagy ne ugrálj!

hu.wikipedia.org/wiki/Statisztik%C3%A1k_az_ateizmusr%C3%B3l_%C3%A9s_vall%C3%A1sr%C3%B3l#V.C3.A9gzetts.C3.A9g_foka

"Hogy a mai emberiség 10% ateista, én kétlem"

értem, tehát te csak a saját statisztikádnak hiszel, a másét le se szarod.

"Viszont a vita során az hangzott el, hogy ha isten tényleg rejtette, akkor miért hiszik annyian, hogy létezik? Az „annyianhoz” elég lenne, ha a mai emberiség fele hinné ezt. De nem a fele hiszi, hanem sokkal több."

a kérdéshez a 10 százalék is elég. az 5 is elég lenne. az emberiségnek például kevesebb, mint 5 százaléka nem hisz mondjuk a saját országa vezetőjének létezésében.

"Ilyen egyebek mellett D’Espagnat (csúcskutató) azon állítása, amely szerint a kvantumelméletből egyenesen következik egy világfenntartó elme/szellem létezése."

de ez nem elfogadott tudományos elmélet, tehát csupán egy kutató elrugaszkodott interpretációja.

"De említheném Spaemann istenérvét is, amely szerint a futurum exactum grammatrikai léte, s az ide vonatkozó ontológiai következmény, posztulálja (= szigorúan megköveteli) Istennek mint egyetemes szellemi ágensnek a létét (ezt most nincs hely kifejteni, de elolvasható)."

kezded lejáratni magad az ilyenekkel.

"Isten léte v. sz. igazolható, bizonyítható, de ezzel csak egy valószínűséghez jutunk"

ha ez igaz volna, akkor elfogadott tudományos elmélet volna isten létezése. ugyanis az igazolt tudományos elméletek pont ezen a szinten vannak. de isten léte tudományosan nem elfogadott, tehát vakítasz.

"Egyébként lebaromozni a másik felet, az nem menő. "

nálam menő a barmokat lebarmozni. én az egyenes beszéd híve vagyok, te az értelmetlen illem híve.

Brendel Mátyás · http://ateistaklub.blog.hu/ 2013.11.25. 20:05:06

@PHILO: nézd, Gödel istenérve ontológiai istenérv. olyan, mint Anzelm istenérve, kicsit kacifántosabb formában. tudjuk, hogy ezt Aquinoi Tamás és Kant sem fogadta el. magyarul Altrichter Ferenc kivesézte a témát.

számomra a dolog ab ovo nevetséges, mint mindenki, aki azt hiszi, bármilyen ontológiai istenérv működhet. ez az elképzelés vészes reality check error jele.

Brendel Mátyás · http://ateistaklub.blog.hu/ 2013.11.25. 20:17:56

@PHILO:

"A kvantumelmélet szerint egy adott elektron adott helyen és adott időben meghatározhatatlanul vagy bizonytalanul részecske vagy hullám természetű."

tévedés. az elektron se nem részecske, se nem hullám. a kvantumelmélet pontosan leírja, hogy milyen. a részecske vagy hullám az a pontatlan hétköznapi megközelítése a dolognak, és ezért van a látszólagos ellentmondás, mert hétköznapi, pontatlan fogalmakkal közelíti meg a nem szakember a szakszerűen egyébként teljesen problémamentes fizikai kérdést. ez olyasmi, mint hülye gyerekeknek elmagyarázni valami bonyolult dolgot. az ő szintjükre levíve a dolog ellentmondásosnak látszódhat. valójában nem az.

"3. Ezt a mélyebb valóságot „leplezett realitásnak” nevezi (veiled reality), amely a tudomány számára csak és kizárólag a kísérletek kapcsán fejeződik ki"

ez a kvantummechanika egy meglehetősen egzotikus, furcsa interpretációja. ha erre támaszkodik az istenérv, akkor kurvára gyenge. miért kéne nekem így interpretálnom a kvantummechanikát?!

"4. Ez a mélyvalóságot mégis felfogjuk, mert ez minden tapasztalat, kísérlet, gondolkodás alapja, a kvantumkísérletek és egységes leírásuk alapja. 5. Ezt a mélyvalóságot érzelmekben, esztétikai élményekben, vallási tapasztalatban sejtjük meg, de nem fogalmilag. 6. Ezt a mélyvalóságot Istennek nevezhetjük."

de nevezhetjük akár babignak is.:)

"8. Mivel az ember nem létezett mindig, sőt a földi élet sem, sőt semmiféle élet sem létezett vagy fog létezni mindig, fel kell tennünk, hogy van „végső megfigyelő”"

maga ez az elképzelés a kvantummechanika koppenhágai értelmezésén alapul, ami már eleve csak egy értelmezés.

en.wikipedia.org/wiki/Interpretations_of_quantum_mechanics

de a koppenhágai értelmezés sem teszi szükségessé, hogy mindig legyen megfigyelő. amíg nincs megfigyelő, addig a koppenhágai értelmezés szerint a jelenségek szuperpozíciós állapotban vannak. nem omlik össze a hullámfüggvény. semmi okunk feltételezni, hogy ez nem lehetett így. de semmi okunk a koppenhágai értelmezéshez sem ragaszkodni.

Brendel Mátyás · http://ateistaklub.blog.hu/ 2013.11.25. 20:19:59

@Koós István:

"nem olvasta Nagelt, meg nem érti. "

egyébként olvastam Nagelt, Chalmers, Dennettet, Churchlandet, tehát a kérdés legfontosabb irodalmát. engem nem lehet átverni a palánkon ezzel sem. Nagelt sokan kritizálták.

Fluctuat 2013.11.25. 21:13:49

Végigolvasva a kommenteket engedjenek meg nekem egy megjegyzést a t. ateista urak. Az a nagyképű, szitkozódó, lehordó, lenéző, megvető stílus, amit különösen Brendel enged meg magának, az ún. "ateizmus" teljes értéktelenségét, hasztalanságát, értelmetlenségét, fanatizmusát, elfajultságát mutatja. Sok magasan kvalifikált tudóssal volt dolgom a világban, de akikkel soha nem találkozhattam, azok is hasonlóak lehettek az írásaik szerint: a kellő modor, mértéket, illemet és visszafogottságot megtartották. érvelésükben nem szofisztikáltak, hanem magyaráztak. Amikor tévedésre hívták fel a figyelmüket, nem agyaraskodtak, hanem bocsánatot kértek. Még a legnagyobbak is, egy Popper vagy egy Schrödinger, akik közül az elsővel volt szerencsém még szót váltani. Teljes mértékben elképzelhetetlen az a pimasz arroqancia, amit ezek, akik még az egyetemi szinten is alig jutottak túl, megengednek maguknak. Sajnos feltehető, hogy az efféle viselkedés mögött valójában súlyos pszichológiai, esetleg pszichiátriai zavarok húzódnak meg, ami az egész ateizmus-mozgalomra sötét árnyékot vet.

Koós István 2013.11.25. 21:28:06

@PHILO:

"4. Ez a mélyvalóságot mégis felfogjuk, mert ez minden tapasztalat, kísérlet, gondolkodás alapja, a kvantumkísérletek és egységes leírásuk alapja. 5. Ezt a mélyvalóságot érzelmekben, esztétikai élményekben, vallási tapasztalatban sejtjük meg, de nem fogalmilag. 6. Ezt a mélyvalóságot Istennek nevezhetjük."

Ez azért durva. A kvantumvalóságot először elkereszteled "mélyvalóságnak", aztán azonosítod azt az esztétikával. Ha te filozófiát végeztél, akkor nyilván tanultál esztétikát. Az esztétikát szerinted lehet a kvantumfizikával azonosítani?

Koós István 2013.11.25. 21:32:25

@PHILO:

Ez tipikusan az analógiás gondolkodás. Ha az elektronnak nem tudjuk egyszerre megmérni a sebességét meg a spinjét is (ha jól emlékszem), akkor az az érzékelésünktől függ, tehát csak a mi tudatunk létezik, innen pedig már vezet az út a misztikus képzelgésekhez.

Olvasd el Eco regényét, a Foucault ingát. Ezt az analógiás ezoterikus vallásos gondolkodást parodizálja ki benne, amely úgy működik, hogy hasonlóságot keresünk a dolgok között, és amit hasonlónak találunk, arról kimondjuk, hogy azonos.

Brendel Mátyás · http://ateistaklub.blog.hu/ 2013.11.25. 21:33:23

@Fluctuat:

"De gustibus non est disputandum". Ha neked nem tetszik ez a stílus, akkor nem tetszik, de ne gondold már azt, hogy ez érv, vagy nekünk ehhez alkalmazkodni kéne!

Én eddig még sose láttam fekete tudóst, de nem gondolom, hogy egy tudós ne lehetne néger. Kissé korlátolt vagy.

Miklós123 2013.11.25. 22:33:54

Nagyon sok irányba mentünk el, és csúnya szitkozódás kezdődött több oldalról is (de azért Brendel Mátyás élenjár benne...).
Koós úrnak:
1. Igen, a szerelem szubjektív dolog, Isten létezése meg objektív. Ámde a vallásosak számára az istenhit egy, a szerelemhez hasonló, szép, felemelő érzés, amely - a szerelemhez hasonlóan - lehetetlenné teszi számukra, hogy tárgyilagosan vizsgálják a valóságot. Megpróbálom őket rávezetni arra, hogy nincs igazuk, de az érzéseiket tisztelem, ezért sohasem vagyok durva.
2. A moralitás nem lehet objektív, mert konszenzuson alapul? Ezt nem egészen értem. A moralitás bizonyos kérdéseiben konszenzus van, de nem azért objektív a moralitás, mert konszenzus van róla, hanem fordítva. A Föld gömb alakú, ez egy tény, és - ma már - konszenzus létezik erre vonatkozóan, de nem azért gömb alakú a Föld, mert az emberek ezt gondolják, hanem azért gondolják így, mert így van és mára felszabadultak azon tévképzetek, előítéletek alól, amelyek az ellenkezőjét hitették el velük.
3. Az általam hozott példa - a magántulajdon szentsége - nem azt jelenti, hogy "tessék, itt van egy esete a vitathatatlan objektív morális szabályoknak", hanem azt próbáltam bemutatni: milyen módon folyhat a morális vita.Egyébként a kommunisták nem a magántulajdont, hanem a termelőeszközök magántulajdonát tartották erkölcstelennek, és - elenyésző számú és rövid ideig tartó kísérletezés után - minden kommunista kísérletben betartották azt az elvet, hogy a személyes tárgyakat igenis az emberek tulajdonolják és azt tiszteletben kell tartani.

Miklós123 2013.11.25. 22:40:12

Mezei Balázs felvetette a vitában, hogy a "van" szó jelentése nem egyértelmű. Isten a vallásos gondolkodók szerint nem úgy "van", mint az empirikus objektumok, nem ellenőrizhető a léte empirikus eszközökkel. Brendel (és a logikai pozitivizmus) azt állítja, hogy a "létezni"-nek nincs más jelentése, mint az empirikus ellenőrizhetőséggel való létezés, és ezesetben az Isten létezése nem egy hamis, hanem egy értelmetlen hipotézis. De a mentális - csak szubjektíven tapasztalható - jelenségek létezése legalábbis felveti azt a problémát, hogy a "létezni" nem azonos az "empirikusan ellenőrizhetően létezni"-vel, hogy talán létezhet valami,ami empirikusan - elvileg sem - igazolható.
Hogyha a mentális jelenségek analizálhatóak valamiféle "epifenoménként", fizikai jelenségeken "szuperveniáló" tényekként és így tovább - szóval ha az agyelméleti materializmus végleg igazolódik, még akkor is ez az azonossága az agynak és a tudatnak csak egy logikailag kontingens azonosság, és még mindig nem zárja ki annak a logikai lehetőségét, hogy létezhet szellem fizikai hordozók nélkül, és akkor Isten is létezhet (bár természetesen ez a hipotézis ellentmond minden tapasztalatunknak, amelyet az Univerzumról szereztünk és így nagyon valószínűtlen).

Miklós123 2013.11.25. 22:44:47

Koós úr szerint valami, ami a valóságra vonatkoztatva "hülyeség", legfeljebb költői alkotásként lehet szép. Nem hiszem. Aquinói SZt. Tamás érdekesen boncolgatta egy sor filozófiai fogalom szerkezetét, mibenlétét (pl. a lét vagy a tudás) ez nem úgy érdekes, vagy szép, vagy hashznos mint egy vers, vagy festmény, inkább úgy, mint egy abztrakt matematikai levezetés.

Brendel Mátyás · http://ateistaklub.blog.hu/ 2013.11.25. 22:53:01

@Miklós123:

"amely - a szerelemhez hasonlóan - lehetetlenné teszi számukra, hogy tárgyilagosan vizsgálják a valóságot. Megpróbálom őket rávezetni arra, hogy nincs igazuk, de az érzéseiket tisztelem, ezért sohasem vagyok durva."

no, ez egy olyan eset, amikor nem lehet megmondani, nem inkább egy nagy pofon-e a hatásos.

"Az általam hozott példa - a magántulajdon szentsége - nem azt jelenti, hogy "tessék, itt van egy esete a vitathatatlan objektív morális szabályoknak", hanem azt próbáltam bemutatni: milyen módon folyhat a morális vita."

ennél fontosabb lenne megmondani, hogy érted az erkölcs objektivitását, és mondani egy példát szerinted objektív erkölcsi elvre. ehelyett itt mellékes dolgokkal foglalkozol. szerintem azért, mert kerülöd a forró kását.

Brendel Mátyás · http://ateistaklub.blog.hu/ 2013.11.25. 22:55:51

@Miklós123:

"ez nem úgy érdekes, vagy szép, vagy hashznos mint egy vers, vagy festmény, inkább úgy, mint egy abztrakt matematikai levezetés. "

a matematika bevallottan nem a valóságról szól, viszont az fontos, hogy a természettudományokban használható. ha nem volna, akkor nem volna fontos, legfeljebb egy hobbi volna, mint a sakk.

Brendel Mátyás · http://ateistaklub.blog.hu/ 2013.11.25. 23:05:33

@Miklós123: egyébként nem értem, mi ez a rögeszméd az érzések tiszteletével. az emberek érzései számomra:

1) vagy rokonszenvesek, akkor nem tisztelem őket, hanem kedvelem.

2) vagy hidegen hagynak, akkor nyilván nem tisztelem, hanem nem érdekel

3) vagy ellenszenvesek. ekkor mivel liberális vagyok, szintén hidegen hagynak, mert elismerem az emberek jogát bármilyen érzésekre

ellenben, ha valaki objektív hülyeséget mond, illetve kárt is okoz nekem, akkor nem érdekel, hogy rokonszenves vagy ellenszenves az érzése, engem az érdekel, hogy nincs igaza, és különösen, hogy kárt okoz.

nem érdekel, hogy valaki mondjuk a családját féltve gyúlöli és írtja a cigányokat. az égadta világon semmit nem mentik az érzései, és nem tisztelem őket.

egyébként a hívők érzéseit általában nem is különösebben szeretem. úgy állok a vallásos érzületekkel, mint Nietzsche. számomra kicsinyes, unalmas, túlságosan is emberi dolog. a negatív értelmében.

számomra a templomok utálatos helyek, kiráz a hideg tőlük, kerülöm őket.

kíváncsi vagyok, ezt az érzést tudod-e tisztelni, vagy csak az inkvizícióhoz, terrorizmushoz és népírátshoz vezető érzéseket tudod.

Miklós123 2013.11.25. 23:34:42

Kedves Mátyás!
Tisztázni kell, igazad van. Sajnos elfogult vagyok. Voltam vallásos - ezt tudod, mert olvastad a letérésem történetét. Nem hiszem el a vallást, azonkívül nem vagyok meggyőződve arról, hogy a vallás hasznos lenne. (Bizonyára sok haszna van: vígaszt ad szerencsétlen embereknek, közösségszervező erő, nagy művészi alkotásokat inspirált, erkölcsre nevel . és bizonyára sok káros hatása: fanatizmus, szexuális elfojtás, a tudatlanság fenntartása az emberekben, én nem tudom megítélni a különböző oldalak súlyát...) A saját egykorvolt vallásosságomról azt gondolom, hogy nekem káros volt (akkor, amikor a kortárs csoportba való - amúgy is nehéz - beilleszkedés lett volna az elvégzendő életfeladatom, bezárt egy olyan közösségbe, amely lényegében nálam két generációval idősebb öregemberek rezervátuma volt). Ámde: számomra a saját vallásos élményeim, érzéseim "szépek" voltak, rokonok a szerelem vagy a nagy művészi alkotások, vagy az emberiséget felemelni akaró nagy eszmék (pl. a kommunizmus) által kiváltott lelkesedéssel, magasztos érzésekkel. Ezt nem csak én érzem így, hanem sok más vallásos vagy valaha vallásos ember is.

Brendel Mátyás · http://ateistaklub.blog.hu/ 2013.11.25. 23:40:41

@Miklós123: Kedves Miklós,

ez rendben, de ugye ehhez az is hozzátartozik, hogy belátod, mástól nem várhatod, hogy ugyanígy érezzen a vallások iránt szerelmet.:)

for 2013.11.26. 02:45:34

Kedves Mindenki!
Azt gondolom, illő hozzászólnom, mivel én írtam a kommentet, ami bejegyzés lett. Igyekeztem hangsúlyozni, hogy ez egy szubjektív beszámoló, de vállalom a letolásokat, mert nem jegyzeteltem. Ezzel együtt nem emlékszem sok egyébre, még a linkelt objektív leírás elolvasása után sem.
Azt is hozzá kell tennem, hogy természetesen jónak és szükségesnek tartom a hasonló rendezvényeket, de olyan formában nem látom értelmüket, hogy veregetjük a valóság elől menekülők vállát.
Mit értek ezen?
Azt, hogy semmilyen hiedelem nem érdemel tiszteletet, épp ahogy az áltudomány és a sarlatánság sem. Ennek muszáj végre világosan látszani, mert a vallásosság amúgy is együtt jár, érzésem szerint, egyfajta felsőbbségtudattal, amit éppen ez a felesleges tisztelet táplál.
Azt, hogy – még ha nem is ismerek egy témát – bármilyen megszólalást kritikusan kezeljek, hivatkozást, magyarázatot vagy pontosítást kérjek rá, legyen szó matematikáról vagy statisztikáról. Eleve, kérdezzek rá olyan dolgokra, amik borzasztóan nem plauzibilisnek vagy cáfolhatónak tűnnek.
Azt, hogy ne fogadjam el, hogy egy objektív dolog létezésének eldöntésére egyenlő eszköz a logikai-empirizmus és a metafizika, mert ebből – mint jelen esetben is – az látszik, mintha egyenlő súlyú érvek állnának egymással szemben. Ráadásul a hallgatók azt érzik, hogy „de jó, a kognitív disszonanciám kiváló feloldása a metafizika” :-).
Azt, hogy amikor valaki a „vanni” változatait emlegeti, akkor muszáj szólni, hogy ez nem értelmes érv, nem értelmes fejtegetés, hanem a lehetséges kiút keresése a konkrét kérdések konkrét megválaszolásának kényszere alól.
Véleményem szerint egy ismeretterjesztő jellegű vitának gondolatébresztőnek kell lennie, és meg kell mutatnia a hallgatóknak, hogy a vizsgált dolog miért úgy van, ahogy gondolom. Ebbe szerintem beletartozik az, hogy az álokoskodást agresszíven, kőkemény érvekkel zúzzuk porrá.
Senkit, főleg Miklóst nem volt szándékom megbántani a véleményemmel, de azt gondolom, elég toleráns volt eddig a vallással mindenki, és ez nem vezetett sehova. A hülyeség, az metafizikai vagy vallásos köntösben is hülyeség.

Koós István 2013.11.26. 08:50:12

@Miklós123:

Nemigen értem, hogy miért kellene tisztelni egy olyan érzést, ami egy elfogadhatatlan, mert érvekkel alá nem támasztható nézethez kapcsolódik.

Egyébként régen én is vallásos voltam, de miután ateista lettem, hosszú ideig én is úgy gondoltam, hogy a vallásos érzéseket tiszteletben kell tartani. Dawkins győzött meg arról, hogy ez hülyeség. Miért kéne? Ha valaki azzal jön, hogy ő kukorica, mint a viccben (lentebb leírom), akkor azt mondom, hogy nem kukorica, de tiszteletben tartom, hogy annak képzeli magát?

A tiszteletet szerintem csak valamilyen teljesítmény érdemli meg, meg persze ott van az ember személyes méltósága, de a tévképzeten nem értem, mit kellene tisztelni. Ezen az alapon minden tévképzetet tisztelni lehet.

Ez azért sem túl rokonszenves nekem, mert a tisztelet sokszor ellehetetleníti a vitát. Mint pl. Eco könyvében, amit egy olasz püspökkel folytatott, ahol a sok udvariaskodás, magyarázkodás megöli a vitát.

A vicc, amire utaltam:

Az ember bekerül a gyogyóba, mert azt képzeli magáról, hogy ő kukorica. Az orvosok hosszú ideig győzködik, hogy ő nem kukorica, hanem ember. Végül megkérdezik tőle a vizsgálatnál:
- Na, mondja, i maga?
- Ember vagyok, doktor úr, most már tudom, hogy nem vagyok kukorica.
Hű, örülnek az orvosok, meggyógyult az ember! Mehet haza a farmra az állatai közé, folytathatja a munkát a gazdaságban. Erre megkérdezi az ember:
- De tessék mondani, doktor úr, otthon a csirkék is tudni fogják, hogy nem vagyok kukorica?

Koós István 2013.11.26. 09:18:33

@Miklós123:

"Koós úr szerint valami, ami a valóságra vonatkoztatva "hülyeség", legfeljebb költői alkotásként lehet szép. Nem hiszem. Aquinói SZt. Tamás érdekesen boncolgatta egy sor filozófiai fogalom szerkezetét, mibenlétét (pl. a lét vagy a tudás) ez nem úgy érdekes, vagy szép, vagy hashznos mint egy vers, vagy festmény, inkább úgy, mint egy abztrakt matematikai levezetés."

Elmondom, mire gondoltam.

A költői nyelv alapvető jellemzője, hogy nincs közvetlen valóságreferenciája. Itt van pl. Rilkétől versrészlet (egy szót sem tudok németül, úgyhogy sajnos csak fordításban ismerem, de ez most mindegy, egyébként Nemes Nagy Ágnes fordítása):

"Én fa vagyok, háttérre rajzolódva,
sok szám közül egy száj vagyok a szóra,
és az, amely elnémul legelébb."

Itt egy olyan lírai én beszél, amely sokszorosan is ellehetetleníti, hogy beszélő, önkifejező szubjektumként képzeljük el. Egyrészt mivel faként nem tudna beszélni, másrészt mivel sok szája van, egy vizuálisan teljesen elképzelhetetlen lényként jelenik meg, és mivel a sok száj sok különböző dolgot mond, a beszéde nem koherens. Ráadásul elnémuló szájként identifikálja önmagát.

Ez a versrészlet tehát éppen a referenciát lehetetleníti el, de közben eléggé szuggesztív ahhoz, hogy ennek ellenére az olvasó mégis megpróbálja elképzelni és megérteni, hogy miről van szó. Nekem ez nagyon tetszik, és nagy nyelvi invencióra utal.

Amikor Szent Tamás azt mondja, hogy Isten nem egy létező, hanem a létaktus totális intenzitása, és Istenben a lét és lényeg nem válik külön, akkor nekem ez is tetszik, de nem mint valami olyan, aminek bármi referenciája lenne, hanem pont úgy, mint a Rilke-szöveg. Ha valaki ezzel akar érvelni Isten léte mellett, akkor azt mondom, hogy hülyeség. De költői teljetményként elfogadható, és értékelhető.

Brendel Mátyás · http://ateistaklub.blog.hu/ 2013.11.26. 09:27:49

@Miklós123 @Koós István:

Még eszembe jutott pár dolog a vallásos érzések tiszteletéről, főleg István legutóbbi hozzászólását olvasva.

1) Egyetértek Istvánnal, ez a tisztelet megöl sok vitát. A könyv,a melyre utal is ilyen, én hallottam pár ilyen vitát, és a beszámolók alapján ez a vita is ilyennek tűnt. Bizony oda kell vágni a hívőknek keményen a tnyelet ás az érveket, és te, Miklós több ilyen lehetőséget kihagytál. Az ATV ismertetése szerint legalábbis még mindig így tűnik.

2) Képzeljünk el egy embert, aki szerelmes Cindy Crawfordba! Elismerjük-e eme embernek az igényét, hogy valaki szeresse? Igen. Ezt az érzését tiszteletben lehet tartani. Tisztelni nem kell. Elismerjük-e eme ember szerelmét Cindy Crawford iránt?! Hát, egye fene, hadd szeresse, ha úgy érez! No de ha ez az ember paranoiás, és elmegy odáig, hogy azt hiszi, Cindy Crawford szereti őt, akkor ezt is tiszteljük?! Szerintem nem. Még mindig eléldegélhet a paranoiájával a maga kis életében. De ha ez az ember elkezd terroristává válni, és valami kárt okozni a társadalomban a képzelt szerelme alapján, akkor nem tisztelem, hanem igen keményen odamondok neki. Itt már az, hogy ő egy szerencsétlen nyomorult, nem tiszteletre méltó, mert károkat okoz másoknak. És a vallás is ilyen.

3) Ezzel szemben tudod, mi az, ami tiszteletre méltó szerintem?! Ha egy ateista, annak ellenére, hogy szinte minden ember fél a haláltól, szinte minden embernek van igénye arra, hogy ne haljon meg, és hogy a világ igazságosabb legyen, a jók elnyerjék jutalmukat, a gonoszok megkapják a büntetésüket, tehát, ha egy ateista ennek ellenére nem lóg mesék tején, hanem bátran szembe mer nézni a valósággal, ezzel az istentelen Univerzummal, és meri azt mondani, hogy ennyi, ezt kell szeretni.

NO ez a tiszteletre méltó és nem a hívők gyávasága. Erről a bátorságról kevés szó esik a közéletben.

Miklós123 2013.11.26. 09:28:29

Kedves Mindenki!
Még egyszer a "tisztelet" kérdéséről: Nem, egy tévképzetet (pl. "Én vagyok Napóleon") vagy egy szimpla babonát ("boszorkány a Mari néni és éjjel lappantyúvá változott és kifejte kecském tejét...") nem kell tisztelni és ha valaki tiszteli, az az ő abszolút szubjektív magánügye. Az itt kommentelők a vallást hasonlónak fogják fel és nem hajlandóak tisztelni.
Én ezzel nem így vagyok. Marx Károlynak mindenki ismeri azt a megállapítását, hogy "a vallás a nép ópiuma", kevesebben ismerik azt, ami előtte áll: "a vallás egy szívtelen világ lelke...". A vallás marxi (illetve feuerbachi, mert onnan vette ő ezt lényegében) kritikájának a lényege az, hogy az "Isten", az "isteni" nem egyéb, mint az emberi lényeg kivetítése, fantasztikus megvalósulása. Az Isten tulajdonságai (szellem, szabadság, szeretet) az ember "nemes", őt az állatok fölé emelő tulajdonságai, amelyek ebben a világban nem valósulhatnak meg, ezért az emberek egy természetfeletti szférába, egy fiktív lényre kivetítve "helyezik" őket a saját világképükben, azért, hogy azt tudják hinni: ezeknek a dolgoknak van valami "helye" a világban, sőt, kitüntetett helye van... Ez illúzió, de szép illúzió... Például az én édesanyám - egyébként egy nagyon szerencsétlen, magányos és beteg ember - rendszeresen imádkozott értem. Szorongott és megnyugtatta őt az, hogy egy gondoskodó "jó Isten" vigyáz az ő fiacskájára... Én nem szeretem a szexuális gátlástalanságot. Kamaszkoromban azért voltam vallásos (többek között), mert a vallás (és a templomban lévő öregemberek) "mondták azt", hogy a szexuális szabadosság helytelen. Könnyebb volt úgy vállalnom a saját "konzervatív"nézeteimet, érzéseimet, hogy azt gondoltam: valami rajtam kívüli, objektív tekintély támasztja alá őket... Ezek illúziók voltak természetesen: senki sem vigyázott rám (magamon kívül), hanem történhetett volna velem mindenféle baj, betegség, meg baleset, és senki sem tilthatja meg az embernek (önmagán kívül), hogy össze-vissza, érzelmek és tisztesség nélkül szexeljen akivel csak tud, és ne tisztelje a nőket, hanem csak szexuális tárgyat lásson bennük, és nagyon sok férfi igy is viselkedik... De szép illúziók...Az édesanya szorongása a gyermekéért, vagy a kamaszfiú ösztönös undora az érzelmek nélküli, gátlástalan szexualitástól szép és tiszteletreméltó érzések, és azok maradnak attól, hogy illúziókkal ideologizálják meg őket. Világosabban már sajnos nem tudok fogalmazni, ha nem győzlek meg titeket, akkor nem győzlek meg... akkor az én tiszteletem a vallás iránt egy szubjektív érzés, és kész.

Brendel Mátyás · http://ateistaklub.blog.hu/ 2013.11.26. 09:48:45

@Miklós123:

"Még egyszer a "tisztelet" kérdéséről: Nem, egy tévképzetet (pl. "Én vagyok Napóleon") vagy egy szimpla babonát ("boszorkány a Mari néni és éjjel lappantyúvá változott és kifejte kecském tejét...") nem kell tisztelni és ha valaki tiszteli, az az ő abszolút szubjektív magánügye. Az itt kommentelők a vallást hasonlónak fogják fel és nem hajlandóak tisztelni."

Itt már az homályos, hogy miben is gondolod te a különbséget, és összemosol dolgokat.

1) Az ateisták egy jelentős része a vallást igazolatlan tévképzetnek fogják fel, és emiatt nem hisznek, illetve ezért kritizálják azt. A különféle vallásokban és az itt említett tévképzetekben kognitív szempontból az a közös, hogy tévképzetek. Hogy igazolatlanok, vagy egyenesen inkonzisztens, lehetetlen képzetek.

2) Amit lentebb fejtegetsz, az egy másik szempont. Nem kognitív szempont, hanem érzelmi igény.

3) Az alapvető kérdés, hogy elfogadhatónak, tiszteletre méltónak tartsunk-e egy érzelmi igény alapú vágyvezérelt gondolkodást?! Ha kinyitod a kaput, hogy ez tiszteletre méltó, akkor az összes előtt kinyitod a kaput.

"Marx Károlynak mindenki ismeri azt a megállapítását, hogy "a vallás a nép ópiuma", kevesebben ismerik azt, ami előtte áll: "a vallás egy szívtelen világ lelke...". A vallás marxi (illetve feuerbachi, mert onnan vette ő ezt lényegében) kritikájának a lényege az, hogy az "Isten", az "isteni" nem egyéb, mint az emberi lényeg kivetítése, fantasztikus megvalósulása. Az Isten tulajdonságai (szellem, szabadság, szeretet) az ember "nemes", őt az állatok fölé emelő tulajdonságai, amelyek ebben a világban nem valósulhatnak meg, ezért az emberek egy természetfeletti szférába, egy fiktív lényre kivetítve "helyezik" őket a saját világképükben, azért, hogy azt tudják hinni: ezeknek a dolgoknak van valami "helye" a világban, sőt, kitüntetett helye van... Ez illúzió, de szép illúzió..."

1) Szerintem Marx és Feuerbach ugyan részben megértő volt a nép ópiumával, de kritizálták azt, tehát végül is azt mondták ennek nincs helye. Viszont megszüntetni úgy gondolták, hogy a társadalom fejlődésével lehet.

2) Azóta a társadalom iszonyat sokat fejlődött. Ma már nem Feuerbach és Marx korában vagyunk, nem a 16 évesen robotoló gyerekmunkások vadkapitalista időszakában, és nem a késői feudális viszonyokban. Ma már megtettük azt a lépést, hogy a vallást, mint a nép ópiumát már átmenetileg sem fogadhatjuk el.

" Például az én édesanyám - egyébként egy nagyon szerencsétlen, magányos és beteg ember - rendszeresen imádkozott értem."

Nézd, aki Napoleonnak képzeli magát, annak is igen sok érzelmi alapja lehet, és igen szerencsétlen ember lehet, hogy idáig jutott, hogy az agya ilyesmikkel téveszti meg magát. És igen, azért, hogy jobban érezze magát. Pont úgy, mint a vallásos hívők.

"Szorongott és megnyugtatta őt az, hogy egy gondoskodó "jó Isten" vigyáz az ő fiacskájára..."

Aki Napoleonnek képzeli magát annak a szorongásán is nyilván sokat enyhít ez a képzet.

"Könnyebb volt úgy vállalnom a saját "konzervatív"nézeteimet, érzéseimet, hogy azt gondoltam: valami rajtam kívüli, objektív tekintély támasztja alá őket..."

Ó ne már, hát ennél ócskább indokot a vallásra! Ha valaki nem akar szabados lenni a szexben, ne legyen, hát nehogy már ebből is gond legyen! Bármelyik társadalomban a legnagyobb probléma nélkül meg lehet élni úgy, hogy az ember, főleg a felnptt ember totál absztinenciát fogad a szexben. Az égadta világon senki nem fog senkit sem zaklatni azért, hogy ugyan már szexeljen egy kicsit, hacsak nem valami őrült szexbomba.

"érzelmek és tisztesség nélkül szexeljen akivel csak tud, és ne tisztelje a nőket, hanem csak szexuális tárgyat lásson bennük, és nagyon sok férfi igy is viselkedik... "

itt aztán eléggé összemosol pár dolgot:

1) a vallásos előírásoknál szabadosabb szex nem feltétlenül szerelem nélküli. ugye a katolikus vallás előírásai szerint például az ember egyszer legyen szerelmes, házasodjon, és csak a feleségével szexeljen, csak házasság után. ezzel szemben az élet realitása, hogy az ember többször tud szerelmes lenni, szakíthat, elválhat, és újra lehet szerelmes.

2) aki ennél is szabadosabb, ott még lehetnek érzelmek, ha nem is szerelem

3) két ember kölcsönös egyetértésben szeretkezik egymással szerelem nélkül, csupán vonzódásból. megmondanád, hogy mi ebben a nők nem tisztelete?!

"De szép illúziók...Az édesanya szorongása a gyermekéért, vagy a kamaszfiú ösztönös undora az érzelmek nélküli, gátlástalan szexualitástól szép és tiszteletreméltó érzések, és azok maradnak attól, hogy illúziókkal ideologizálják meg őket. "

Szerintem egyáltalán nem tiszteletreméltó az, hogy szerelmi és szexuális téren egy viszonylag korlátolt, tájékozatlan, tapasztalatlan képzeted van. Felőlem te érezheted így, élhetsz így, csak azt ne gondold már, hogy nekünk ezt tisztelni kell!

Brendel Mátyás · http://ateistaklub.blog.hu/ 2013.11.26. 09:54:46

@Miklós123:

Őszintén szólva én az életemben sok nővel szeretkeztem, volt, akivel szerelemből, volt, akivel vonzódásból. Mindig kölcsönös egyetértésben. Semmi gondom nincs ezzel, vállalom, szerintem vállalható. Ha visszagondolok az eddigi életemben, igazán nem szeretném, ha ehelyett valami korlátolt szerelmi és szexuális életem lett volna.

Ha a hívők máshogy akarnak élni, abba nem szólok bele. De a jellemző az, hogy ők akarnak beleszólni abba, mi hogy éljünk. És ez nem tiszteletre méltő, hanem felháborító és sértő. És kikérem magamnak. Pont ez az egyik nagy gond ma a vallással. És pont ezért zuhan az egyházak és a vallások híveinek száma. Ezt nem fogják fel.

Ha pedig te ilyen régimódiságból szimpatizálsz a vallásokkal, akkor egyrészt én nem szólok bele, milyen életet élsz, felőlem tiszteld a vallást ezért. Valóban sok közös van bennetek.

De érdemes észrevenni, hogy te nem a mainstream ateista vagy, ne várd el, hogy az ateisták többsége veled együtt érezzen. Te egy fura ateista vagy, ha egyáltalán igazán ateista vagy, és nem a többséget képviseled.

Számunkra a te érzelmeid azt is jelentik, hogy például egy ilyen vitában nem képviseled az ateizmust azzal az erővel, ahogy azt képviselni lehetne.

Emiatt a cím végül is jogos. A vallásoknak például szexualitásban nem veled kell vitatkozniuk, hanem a modern mainstream felfogás képviselőivel, akik fiatalok tömegeit képviselik. Veled hiába vitáznak, akár meg is egyezhettek, kezet is foghattok, a fiatalok tömegeit ettől még el fogják veszíteni.

Miklós123 2013.11.26. 09:56:54

Brendel Mátyásnak igaza van abban, hogy amit a vallás nélküli ámde objektív moralitásról mondtam, az nem elég világos. Nem akartam a forró kását kerülgetni, hanem csak nem akartam itt túl terjedelmes elemzésekbe belefogni.
Megpróbálok világos lenni hát:
Azt gondolom, az erkölcs minden társadalomban betölt egy bizonyos funkciót, ugyanúgy, mint az oktatás. (Minden társadalomban nevelik valahogy a gyerekeket, megtanítják nekik az adott társadalomban való éléshez szükséges ismereteket készségeket. És minden társadalomban van valamilyen szabályrendszer, amely határt szab az emberek ösztönös illetve érdek-vezérelt cselekvéseinek, azért, hogy a közösség együtt tudjon élni.
Mivel az erkölcsnek van egy bizonyos funkciója, racionálisan lehet vitatkozni arról, hogy milyen erkölcs töltheti be "jól" ezt a funkciót és milyen nem. Kant megragadta például az erkölcsnek egy alapvető (talán nem az egyetlen...) vonását: azt, hogy az erkölcs univerzalizálható kell hogy legyen, hogy az erkölcsi szabályok olyanok kell, hogy legyenek (különben nem működhetnének az életben), hogy minden ember - konkrét helyzetétől és érzéseitől függetlenül - be tudja tartani őket. Ezért nem lehet olyan szabályokat elfogadni, amelyek egyetemesen alkalmazhatatlanok (pl. "ne tartsd be az ígéreteket" - mert akkor ígéretet sem tudna senki tenni, ha nem feltételezné, hogy az ígéreteket betartják az emberek..). De nem biztos, hogy az univerzalizálhatóság az erkölcs egyetlen kritériuma, lehet, hogy vannak mások is - például az, hogy az erkölcs valamiképpen összefügg az emberek javával, érdekeivel.
Amikor találkozunk a történelemben és a különböző kultúrákban létező egymástól radikálisan különböző erkölcsi felfogások sokaságával akkor úgy tűnhet, hogy az erkölcsi viták eldönthetetlenek. De nem biztos, hogy ez így van. Azoknak az embereknek, akiknek látszólag egy "más" erkölcse van, mint miénk, lehet, hogy vannak a mieinkkel közös "mélyebb", alapvetőbb szabályai és akkor vitatkozhatunk arról, hogy nem lehet-e, hogy nekünk van igazunk, mert mi alkalmazzuk következetesebben és önkényes további előfeltevések nélkül ezeket az alapvetőbb szabályokat. Például az amerikai Délen az emberek százötven éve úgy gondolták - általában - hogy minden gondolkodó, felnőtt embernek joga van ahhoz, hogy szabadon keresse a boldogságot, olyan életformát, társat, lakóhelyet, hivatást válasszon, amilyent akar. Ezt mi is így gondoljuk. A különbség a két felfogás között az, hogy ők úgy gondolták: ez nem vonatkozik a feketékre. Ez vagy azzal támasztották alá, hogy a feketék nem felnőtt emberek, hanem olyanok, mint az állatok, vagy a gyerekek, és ezért nincs ugyanahhoz joguk, amihez a fehéreknek, vagy azzal, hogy a feketék tök jól érzik magukat rabszolgaként. De az első érv egy tudományosan igazolhatatlan, téves állítás, a második pedig egy tarthatatlan premisszát involvál: hogy nekem rajtam kívül valaki más ("a társadalom") mondhatja meg, hogy nekem mi a boldogság, én hogy érezzem jól magam.
Az,hogy a vita racionális érvekkel folytatható, nem jelenti azt, hogy minden erkölcsi kérdés eldönthető észérvekkel. De nincs szó arról, hogy ez a vita eleve reménytelen lenne, hogy egy észérvekkel eleve eldönthetetlen kérdésről folyna, hogy az erkölcs szubjektív érzések, vagy kulturális hagyományok dolga volna. Ilyen értelemben hiszek én egy objektív moralitásban.

Brendel Mátyás · http://ateistaklub.blog.hu/ 2013.11.26. 10:17:08

@Miklós123:

még egy pár dolog:

1) adott esetben egy nő kívánhatja azt, hogy szexuális tárgynak nézzék. akár azt is, hogy csak szexuális tárgynak nézzék. ahogy idősödik, egyre inkább elő szokott ez fordulni.

2) ahog egy férfi is kívánhatja ezt, és ritkán szokott megadatni neki.:) sok férfinak egyáltalán nem.

3) ebben a kérdésben még a Sex and the City c. sorozat bölcsebb nálad, és az összes keresztény, zsidó és muszlim egyháznál.

4) ha valami egy nő számára fontos lenne megtanulni, az az, hogy tudja, mit akar, tudja, hogy meg kívánja-e valósítani, ismerje a következményeket, tudjon okosan dönteni, kormányozza jól szexuálsi életét

5) ebben nem lehet megúszni azt, hogy kísérletezzen, szerezzen tapasztalatot. nincs az a könyv, amely ebben kész megoldást tud neki adni. de ugyanígy férfinek sem.

6) ha viszont könyvhöz fordul valaki, akkor "az istenért" csak ne a Bibliához! A Bibliából egy nő nem fogja megtanulni azt, hogy mi jó neki, nem fogja megérteni, hogy miért az jó neki, miért érdemes így vagy úgy eljárni, mitől füg ez. A Bibliából csak ostoba szabályokat fog megérteni, nem összefüggéseket.

7) A Bibliából egy férfi ostoba tiltásokat fog megtanulni, de nem összefüggéseket. A nők tiszteletét végképp nem. a Biblia egy erősen nőgyűlölő mű.

8) EGyébként egy férfinek még inkább azt tanácsolom, hogy kísérletezzen, majd meglátja. Adott esetben mondjuk egy kis nőtiszteletet lehet neki tanítani, de nem a vallásokból. Ha valamiért le lehet nézni a vallásokat, akkor már csak ezért is. Pont azt nem csinálják, amit szem előtt tartasz: nem tisztelik a nőket. Legalábbis a keresztény és sizlám vallások.

9) Ha egy férfi eleve tisztelni akarja a nőket, ahogy te mondtad, az égadta világon semmi erkölcsi parancsolatra nincs szüksége. Ha valamire szüksége van, az inkább lazítás lehet.

Miklós123 2013.11.26. 11:36:03

Kedves Mátyás!
Sajnos nem értjük egymást. Nem arról van szó, hogy most én szexuális kérdésekben konzervatív vagyok és azt gondolom, hogy azért kell a valláshoz fordulni, mert a konzervatív szexuális morált támasztja alá.
Hanem arról, hogy kamaszkoromban az voltam és ezért szimpatizáltam a vallással. Úgy képzeltem, hogy "magasabbrendű értékek = régimódiság (mert ugye a "mai világban" csak drog, bűnözés, pornó, hülye tévéműsorok, agresszív, kemény zenék, érthetetlen, formabontó irodalmi művek vannak), és "régimódiság = vallás", mert az embereket csak a vallás képes megóvni a haladás káros hatásaitól.
Ez - természetesen - egy baromság. Ilyet csak egy zavart lelkű, és az élettől teljesen elzárt magányos tinédzser tud kitalálni.
Nem kell, hogy ezzel vitatkozzál, nem kell, hogy bizonyítsd nekem, hogy azért, mert valaki nem a durva, érzelemmentes, a nőket nem tisztelő típusú szexualitás híve, még nem kell, hogy a konzervatív szexuális erkölcs híve legyen, és attól, hogy a konzervatív erkölcs híve, nem kell, hogy vallásos legyen. Ezt én is tudom.
Amit mondani akartam az csak, az, hogy engem, és sok más vallásos embert a magasabb rendű értékek (szeretet, spiritualitás, stb.) kultusza, az azokhoz való vonzódás vezetett vagy vezet a valláshoz. Mivel én ezt is látom a vallásban, és nem pusztán csak a szokás és a tudatlanság által deformált emberi gondolkodást, ezért tisztelem a vallást és nem abban a stílusban vitatkozom vele, amelyben te.

Brendel Mátyás · http://ateistaklub.blog.hu/ 2013.11.26. 13:05:37

@Miklós123: na jó, de ez egy olyan kis jelentőségű, apró dolog, mitnha valaki azzal jönne, hogy hát azért lett náci, mert őt a német kultúra nagysága vezérelte (ld. Heidegger).

movie 2013.11.26. 23:13:45

@Brendel Mátyás: Nem találod viccesnek, hogy minden ateistától elvárod a teisták lenézését? Azok akik hisznek, érdemtelenek bármilyen tiszteletre?
Nézz egy kicsit magadba ember, mert szerintem a ló túloldalán vagy!

Brendel Mátyás · http://ateistaklub.blog.hu/ 2013.11.27. 05:57:12

@movie: "Nem találod viccesnek, hogy minden ateistától elvárod a teisták lenézését? "

HAZUDSZ. nem várom el. sehol nem írtam, hogy elvárom. a következő hazugságnál repülsz. ezen a blogon sok mindent lehet, de hazudni arról hogy ki mit írt, azt nem. én nem fogok veled arról vitát nyitni, hogy te megrágalmazol olyannal, amit nem is írtam, és akkor védekezzek.

movie 2013.11.27. 08:10:33

Annyiszor repültem én már, hogy hozzászoktam. Azt is megszoktam, hogy lekezelő vagy velem és mindenkivel, aki nem pont azt vallja amit te.
Én megfogalmaztam a mentalitást amit képviselsz. Betűvel valóban nem írtad le, de a soraid között ez van. Minden írásodból az süt, hogy a teisták annyira hülyék, hogy csak na.

tudati 2013.11.27. 11:36:54

a kvantum kísérlet kizárólag annyiról szól hogy

a két rés kísérlet során az elektron hullámként viselkedik és egy ennek megfelelő mintát hoz létre a túlsó falon

ha pedig meg akarják nézni melyik résen megy keresztül az elektron, akkor már nem hullámként viselkedik a túlsó falon, hanem csak sima össze-vissza mintha lesz, mint amikor nem hullámként viselkedő nagy darab részecskéket lőnek ki

www.youtube.com/watch?v=LbubPpizIwg

vagyis ha nem figyelik akkor a vízhez hasonlóan hullámként viselkedik, ha pedig ránéznek akkor pedig részecskeként

az ezoterikusok és sokan mások ezt félreértelmezik, és úgy fogják fel hogy a gondolattal a kedvük szerint irányíthatják a részecskéket, pedig a kísérletben nincs szó semmilyen irányításról, csak arról van szó hogy hullám tulajdonság van addig ameddig nem néznek rá

és könnyen lehet hogy ez a kikerült tudományos kísérlet sem úgy igaz ahogy elmondják, lehet hogy csak simán a megfigyelésnél követnek el olyan hibát ami tönkre vágja az elektron hullám jellegét, vagy éppen komoly technológiai verseny van, ezért lehet szándékosan fals információt adnak ki, a világon nagyon kevés olyan eszköz van amivel ellenőrizni lehetne a dolgot, ha akarják simán beszopatják a világot

a videó úgy magyarázza hogy szerintük az elektron gondol egyet és ketté válik aztán úgy megy tovább
szerintem reálisabb az hogy az elektronnak van egy energia mezője ami van olyan széles mint a két rés, ezért úgy megy át a két résen mint amikor a víz megy át rajta és emiatt behullámzik az energia mezője

Brendel Mátyás · http://ateistaklub.blog.hu/ 2013.11.27. 12:36:26

@movie:

A) "minden ateistától elvárod a teisták lenézését"

B) "lekezelő vagy velem és mindenkivel, aki nem pont azt vallja amit te."

1) A és B kurvára különböző dolog.
2) A sem igaz, és B sem
3) Félrevezetően, ostobán rágalmazol össze-vissza, bevallottan a tényektől függetlenül.

Ez lenne a keresztény mentalitás?!

Brendel Mátyás · http://ateistaklub.blog.hu/ 2013.11.27. 12:38:42

@tudati:

"a világon nagyon kevés olyan eszköz van amivel ellenőrizni lehetne a dolgot, ha akarják simán beszopatják a világot"

nagyon sok van a világ különböző laboratórumaiban különböző országokban. ráéadásul ez a dolgo már vagy 100 éves, különböző demokráciák, diktatúrák, laboratórumaiban látták ugyanazt a jelenséget.

ne gyere ilyen baromságokkal!

robert81 2013.11.27. 14:48:40

Tisztelt kommentelők! Végigolvastam a vitát, és meggyőzőnek tartom az ateisták érveit. Egyvalamivel azonban nem értek egyet.

@Brendel Mátyás "baromság. nem lehetséges szintetikus a priori."

Ez szerintem tévedés: szintetikus a priori pl. a logikában az ellentmondásmentesség elve: ~(A&~A)

Brendel Mátyás · http://ateistaklub.blog.hu/ 2013.11.27. 15:14:58

@robert81:

" a logikában az ellentmondásmentesség elve: ~(A&~A)"

a logika analitikus a priori

robert81 2013.11.27. 17:15:33

@Brendel Mátyás: "a logika analitikus a priori"

Nem igaz, mert egyes alternatív (parakonzisztens) logikai rendszerek nem fogadják el az ellentmondásmentesség elvét, tehát ez az elv nem lehet analitikus. Más alternatív logikák meg mást utasítanak el, például az intuicionista logika a kizárt harmadik törvényét. Tehát az, amit "klasszikus logika"-ként ismerünk, nem analitikus, hanem szintetikus.

en.wikipedia.org/wiki/Paraconsistent_logic

Brendel Mátyás · http://ateistaklub.blog.hu/ 2013.11.27. 17:24:16

@robert81:

"logikai rendszerek nem fogadják el az ellentmondásmentesség elvét, tehát ez az elv nem lehet analitikus"

ezért analitikus, és pont ezért nem lehet szintetikus.

"Carnap then proceeded to define some kinds of statements: (i) a statement is L-true if and only if it is a logical consequence of the empty set of statements; (ii) a statement is L-false if and only if all statements are a logical consequence of it; (iii) a statement is analytic if and only if it is L-true or L-false; (iv) a statement is synthetic if and only if is not analytic. "

www.iep.utm.edu/carnap/#H3

továbbá a szintetikus állítások pont olyanok, hogy nem lehetnek így is, meg úgy is. a valóságról szóló állítások pláne nem.

ha isten bizonyítása olyan volna, mint az ellentmondásmentesség elve, akkor lehetséges volna az is, hogy isten van, és az is, hogy nincs, csak konvenció kérdése volna. ha isten léte csak konvenció volna, akkor nyilván nem valóságos létezésről szólna a dolog. tehát baromság lenne. egyébként az ntológiaiistenbizonyítás baromság, és például pontosan így baromság.

robert81 2013.11.27. 17:42:53

@Brendel Mátyás: Abból, amit idéztél, épp az derül ki, hogy Carnap analitikusság-definíciója használhatatlan. Az ellentmondásmentesség elvének tagadása a klasszikus logikában (Carnap definíciója szerint) L-hamis. Tehát itt Carnap definíciója alapján az elv maga L-igaz, ezért analitikus. A parakonzisztens logikákban viszont maga az elv a definíció szerint se nem L-igaz,se nem L-hamis, vagyis szintetikus. Akkor most mit mondjunk róla, analitikus vagy szintetikus? Ebből látszik, hogy a kérdésnek csak logikai rendszerhez viszonyítva van értelme.

Brendel Mátyás · http://ateistaklub.blog.hu/ 2013.11.27. 18:04:30

@robert81: Carnap rendszerében az axiómáék nyilván analitikusak, mert L-igazak. ezen kívül analitikus, ami még levezethető belőlük.

Na most egy parakonzistzens rendszerben vagy van levezethetés, akkor Carnap definíciója alkalmazható.

Vagy nincs, akkor meg csak az axiómák lesznek analitikusak.

Nincs itt semmi gond. És messzire kerültünk attól,h ogy az ontológiai istenbizonyítás baromság, mert nincs szintetikus a priori.

A dolognak köze nincs a parakonzisztens logikához.

Miklós123 2013.12.01. 23:22:01

Itten egyes vallással szimpatizálók bedobták a "kvantummechanikát", a kvantummechanika értelmezését, és azt, hogy ott nem lehet az objektív történést elválasztani a szubjektív megfigyeléstől. És hogyha nincs egy objektív anyagi valóság, akkor ugye biztos van valami totális emberfeletti szubjektum, Isten vagy ilyesmi.
Ezek a dolgok már nagyon régiek, évtizedek óta verklizik őket, el is keresztelték ezt a marxisták némi gúnnyal "fizikai idealizmusnak".
Azt hiszem, ezzel kapcsolatban nem árt felidézni Lenint, aki az akkor a tudomány (fizika) eredményeit a szubjektív idealizmus alátámasztására felhasználó fizikus-filozófusnak, E. Machnak felvetette a következő kérdéseket ("Materializmus és empirikriticizmus"):
1. Agyával gondolkozik-e az ember?
2. Létezett-e természet az ember előtt?
Ha "igen"-nel válaszolunk ezekre a kérdésekre, akkor teljesen nyilvánvaló, hogy az emberi megismerés egy pici része a valóságnak, amelyet egy tőle független,az ő létrejöttét évmilliárdokkal megelőző, nála összehasonlíthatatlanul nagyobb rendszer, valóság alakított ki és határoz meg. Ezért a megismerésnek, a szubjektumnak semmiféle befolyása sem lehet az objektív valóságra, és ha egy tudományos elméletből az ellenkezője látszik következni, akkor a tudományos elméletet kell elvetni és nem a filozófiai realizmust.

Brendel Mátyás · http://ateistaklub.blog.hu/ 2013.12.02. 07:01:16

@Miklós123: mondhatják azt, hogy előtte a természet szuperponált állapotban létezett, és az ember döntötte össze a hullámfüggvényeket. cseles dolog, mert minden tök ekvivalens a másik értelmezéssel, azaz ez egy tökéletesen metafizikai elgondolás. olyasmi, mint, Russell példája arról, hogy a világ 5 perce keletkezett, de évmilliárdos történelem nyomaival együtt
süti beállítások módosítása